Sie sind auf Seite 1von 151

ELEMENTARY SET THEORY,

LINEAR AND INTRODUCTORY


ABSTRACT ALGEBRA
SMTH012
Study Guide
“Mathematics is the language with which God
has written the universe.” Galileo Galilei

June 27, 2019


Contents

1 MATHEMATICAL LOGIC 2
1.1 Propositional Logic . . . . . . . . . . . . . . . . . . . . . . . . . . 2
1.2 Conditional and Biconditional Statements . . . . . . . . . . . . . 7
1.3 Laws of Logic . . . . . . . . . . . . . . . . . . . . . . . . . . . . . 10
1.4 Arguments . . . . . . . . . . . . . . . . . . . . . . . . . . . . . . 13
1.5 Predicates . . . . . . . . . . . . . . . . . . . . . . . . . . . . . . . 20
1.6 Negation of Quantified statements . . . . . . . . . . . . . . . . . 22
1.7 Methods of Proof . . . . . . . . . . . . . . . . . . . . . . . . . . . 23
1.7.1 Direct proof . . . . . . . . . . . . . . . . . . . . . . . . . . 24
1.7.2 Indirect proof . . . . . . . . . . . . . . . . . . . . . . . . . 25
1.7.3 Proof by cases . . . . . . . . . . . . . . . . . . . . . . . . 28

2 SET THEORY AND RELATION 30


2.1 Sets . . . . . . . . . . . . . . . . . . . . . . . . . . . . . . . . . . 30
2.1.1 Listing of elements . . . . . . . . . . . . . . . . . . . . . . 31
2.1.2 Important sets of numbers: . . . . . . . . . . . . . . . . . 31
2.1.3 Subsets . . . . . . . . . . . . . . . . . . . . . . . . . . . . 33
2.1.4 Equal sets . . . . . . . . . . . . . . . . . . . . . . . . . . . 33
2.1.5 The empty set . . . . . . . . . . . . . . . . . . . . . . . . 33
2.1.6 Set Operations . . . . . . . . . . . . . . . . . . . . . . . . 34
2.2 Set Algebra . . . . . . . . . . . . . . . . . . . . . . . . . . . . . . 36
2.2.1 Cartesian Products . . . . . . . . . . . . . . . . . . . . . . 40

i
2.2.2 Power Set . . . . . . . . . . . . . . . . . . . . . . . . . . . 43
2.2.3 Symmetric difference . . . . . . . . . . . . . . . . . . . . . 45
2.3 Relations . . . . . . . . . . . . . . . . . . . . . . . . . . . . . . . 46
2.3.1 Types of Relations . . . . . . . . . . . . . . . . . . . . . . 46
2.3.2 Equivalence Classes and Partitions . . . . . . . . . . . . . 50
2.3.3 Congruence Modulo . . . . . . . . . . . . . . . . . . . . . 51
2.4 Algebraic structures . . . . . . . . . . . . . . . . . . . . . . . . . 55
2.4.1 Associativity, Commutativity and idempotency . . . . . . 56
2.4.2 Algebraic structures . . . . . . . . . . . . . . . . . . . . . 57
2.4.3 Examples of algebraic structures . . . . . . . . . . . . . . 57
2.4.4 Boolean algebras . . . . . . . . . . . . . . . . . . . . . . . 60
2.5 Functions . . . . . . . . . . . . . . . . . . . . . . . . . . . . . . . 61
2.5.1 One-to-One and Onto functions . . . . . . . . . . . . . . . 62
2.5.2 Inequalities and absolute value . . . . . . . . . . . . . . . 64

3 MATHEMATICAL INDUCTION, BINOMIAL THEOREM AND


COMBINATORICS 68
3.1 Mathematical Induction . . . . . . . . . . . . . . . . . . . . . . . 68
3.2 Factorials and Binomial Expansions . . . . . . . . . . . . . . . . 72
3.2.1 Binomial expansions and Binomial theorem . . . . . . . . 73
3.2.2 Expansion of (a + b)n when n negative integer . . . . . . 80
n
3.2.3 Expansion of (a + b) when n is a fraction . . . . . . . . . 81
3.3 Combination and Permutations . . . . . . . . . . . . . . . . . . . 82
3.3.1 Permutations . . . . . . . . . . . . . . . . . . . . . . . . . 82
3.3.2 Combination . . . . . . . . . . . . . . . . . . . . . . . . . 85

4 COMPLEX NUMBERS 89
4.1 Operations and Properties of Complex Numbers . . . . . . . . . 89
4.1.1 Operations with complex numbers . . . . . . . . . . . . . 90
4.2 Modulus-Argument form (Polar form) of complex numbers . . . 96
4.3 Euler’s Formula (Exponential Form) . . . . . . . . . . . . . . . . 101

ii
4.4 De Moivre’s Theorem . . . . . . . . . . . . . . . . . . . . . . . . 103
4.5 Finding roots of Complex Numbers . . . . . . . . . . . . . . . . . 106

5 INTRODUCTION TO MATRICES AND LINEAR EQUATIONS111


5.1 Matrix Theory . . . . . . . . . . . . . . . . . . . . . . . . . . . . 111
5.1.1 Matrix Addition and Subtraction . . . . . . . . . . . . . . 114
5.1.2 Matrix Multiplication . . . . . . . . . . . . . . . . . . . . 114
5.2 Echelon Matrices . . . . . . . . . . . . . . . . . . . . . . . . . . . 117
5.3 Determinant . . . . . . . . . . . . . . . . . . . . . . . . . . . . . . 120
5.4 Matrix Inversion . . . . . . . . . . . . . . . . . . . . . . . . . . . 128
5.5 System of Linear equations: Matrix method . . . . . . . . . . . . 134
5.5.1 Matrix Method . . . . . . . . . . . . . . . . . . . . . . . . 135
5.6 3 × 3 systems with parameters . . . . . . . . . . . . . . . . . . . 142
5.6.1 Elementary Matrices and Test for Invertibility . . . . . . 146

1
Chapter 1

MATHEMATICAL LOGIC

“Pure mathematics is, in its way, the poetry of logical ideas”. Albert Einstein

In this chapter we will engage in Mathematical reasoning in order to read,


comprehend and construct Mathematical arguments. The main goal is to un-
derstand how to construct correct Mathematical arguments.

1.1 Propositional Logic


Definition 1.1.1. Proposition:Proposition is a declarative sentence that can
be classified as true or false, but not both.
We will use letter to denote the proposition e.g. P,Q,R etc.

Example 1.1.2. 1. 55 − 2(5) = 0

2. Durban is next to Indian ocean.

3. Steve Biko was the president of South Africa.

4. Bujumbura is the capital city of Burundi.

Proposition 2 and 4 are true, whereas 1 and 3 are false.


Sentences that are not propositions include questions and commands.

2
Example 1.1.3. 5. Go there!

6. Is Martin a girl?

7. x + y = 3

8. 5x = 7

Sentences 5 and 6 are not proposition since they are command and question
respectively, where as sentences 7 and 8 are not proposition since they are
neither true nor false. If we assign values on variables in 7 and 8, the sentences
become proposition.
The truthfulness or falsity of a statement is called its truth value. “1” will stand
for a true statement and “0” for a false statement.

Definition 1.1.4. Compound statements are formed by combining two or


more simple sentences. We use connectives (or logical operators) to com-
bine such sentences. These connectives are “and”, “or”, “not”, “If. . . then . . . ”
and “. . . if and only if . . . ”.

Definition 1.1.5. (Conjunction): Let P and Q be any two statements. The


compound statement “P and Q” is called the conjunction of the statement
P and statement Q. Symbolically we denote this by P ∧ Q.

Example 1.1.6. Let P and Q be statements:

(a) P : Trevor goes to Church.


Q: 8 × 2 − 6 = −10.
Then the conjunction of P and Q is
P ∧ Q: Trevor goes to Church and 8 × 2 − 6 = −10.

(b) P : Square root is defined in the domain of all real numbers.


Q: Lagos city is in France.
Then the conjunction of P and Q is
P ∧ Q: Square root is defined in the domain of all real numbers and Lagos

3
is in France.

The truth value of P ∧ Q depends on the truth values of constituent statements


P and Q. Note that P may be true or false and for each of these Q may be true
or false. So there is a combination of four possibilities. We will summarize the
possibilities by using a truth table.
We can illustrate the truth table of P ∧ Q as follows:

P Q P ∧Q
1 1 1
1 0 0
0 1 0
0 0 0

NB:The conjunction P ∧ Q is true only if both P and Q are true.

Definition 1.1.7. (Disjunction): Let P and Q be any two statements. The


compound statement “P or Q” is called the disjunction of the statement P
and statement Q. Symbolically we denote this by P ∨ Q.

Example 1.1.8. Let P and Q be statements:

(a) P : Trevor goes to Church.


Q: 8 × 2 − 6 = −10.
Then the disjunction of P and Q is
P ∨ Q: Trevor goes to Church or 8 × 2 − 6 = −10.

(b) P : Square root is defined in the domain of all real numbers.


Q: Lagos is in France.
Then the conjunction of P and Q is
P ∨ Q: Square root is defined in the domain of all real numbers or Lagos
is in France.

4
The truth value of P ∨ Q also depends on the truth values of constituent state-
ments P and Q.
We can illustrate the truth table of P ∨ Q as follows:

P Q P ∨Q
1 1 1
1 0 1
0 1 1
0 0 0

NB:

(i) The truth value of P ∨ Q is F only when P is F and Q is F .

(ii) The truth value of P ∨ Q is T when at least one statement is true.

Definition 1.1.9. (Negation): Let P be any statement. The negation of


statement P is the statement “it is false that P” or “not P”. This is
denoted by ¬P .

It is very clear that if P is true, then ¬P is false and vice versa.


We can illustrate the truth table as follows:

P ¬P
1 0
0 1

Example 1.1.10. Let P and Q be statements:

(a) P : Trevor goes to Church.


¬P : Trevor does not go to Church.

(b) P : Gauteng is in Lesotho.


¬P : Gauteng is not in Lesotho.

5
(c) P : 3 < 2 − 5.
¬P : 3 ≮ 2 − 5.
This is equivalent as saying 3 = 2 − 5 or 3 ≥ 2 − 5.

Quite often we will combine the compound statements to obtain more compound
statements. We will see more examples as we go on but we can first do the
following examples:

Example 1.1.11. Construct the truth table of each of the following statements:

(a) (P ∨ ¬Q) ∧ ¬P

(b) ¬(P ∧ Q) ∨ ¬Q

(c) (P ∧ Q) ∨ ¬(P ∧ ¬R)

Solutions

P Q ¬P ¬Q P ∨ ¬Q (P ∨ ¬Q) ∧ ¬P
1 1 0 0 1 0
(a) 1 0 0 1 1 0
0 1 1 0 0 0
0 0 1 1 1 1

P Q ¬Q P ∧Q ¬(P ∧ Q) ¬(P ∧ Q) ∨ ¬Q
1 1 0 1 0 0
(b) 1 0 1 0 1 1
0 1 0 0 1 1
0 0 1 0 1 1

(c)

6
P Q R ¬R P ∧Q P ∧ ¬R ¬(P ∧ ¬R) (P ∧ Q) ∨ ¬(P ∧ ¬R)
1 1 1 0 1 0 1 1
1 1 0 1 1 1 0 1
1 0 1 0 0 0 1 1
1 0 0 1 0 1 0 0
0 1 1 0 0 0 1 1
0 1 0 1 0 0 1 1
0 0 1 0 0 0 1 1
0 0 0 1 0 0 1 1

Tutorials 1.1.12. Construct the truth table of the following statements:

(a) (P ∨ ¬Q) ∧ ¬Q

(b) (¬P ∧ ¬Q) ∨ ¬(P ∨ ¬Q)

(c) ¬(P ∧ Q) ∨ ¬R.

(d) (P ∨ ¬R) ∧ ¬(Q ∨ R)

1.2 Conditional and Biconditional Statements


Definition 1.2.1. (Conditional Statement): Let P and Q be any two state-
ments. The compound statement “If P then Q” is called the conditional state-
ment or an implication. Symbolically we denote this by P ⇒ Q which will read
“P implies Q”. P is called the hypothesis and Q is called the conclusion.

Example 1.2.2. Let P and Q be statements:

(a) P : Trevor goes to Church.


Q: 8 × 2 − 6 = −10.
Then the conditional statement of P and Q is
P ⇒ Q: If Trevor goes to Church then 8 × 2 − 6 = −10.

7
(b) P : A function is continuous on [a, b].
Q: A function is differentiable on (a, b).
Then P ⇒ Q: If a function is continuous on [a, b] then it is differentiable
on (a, b).

(c) P : 4 − 9 ≥ 2.
Q: University of Limpopo is in Cape Town.
Then P ⇒ Q: If 4 − 9 ≥ 2 then University of Limpopo is in Cape Town.

The truth value of P ⇒ Q also depends on the truth values of constituent


statements P and Q.
We can illustrate the truth table of P ⇒ Q as follows:

P Q P ⇒Q
1 1 1
1 0 0
0 1 1
0 0 1

NB:Implication is false only when P is true and Q is false. This means that it is
not possible to deduce a false statement from a true statement, while reasoning
correctly. But it is possible to deduce a true statement from a false statement.

Converse, Contrapositive and Inverse

We can form some new conditional statements starting with a conditional state-
ment P ⇒ Q. In particular, there are three related conditional statements that
occur so often that they have special names:

(a) Converse: Q ⇒ P is converse of P ⇒ Q,

(b) Contrapositive:¬Q ⇒ ¬P is the contrapositive of P ⇒ Q,

(c) Inverse:¬P ⇒ ¬Q is the inverse of P ⇒ Q.

8
We may try to find out which of the above implications have the same meaning.
We will construct the truth table to see this.

P Q ¬P ¬Q P ⇒Q Q⇒P ¬Q ⇒ ¬P ¬P ⇒ ¬Q
1 1 0 0 1 1 1 1
1 0 0 1 0 1 0 1
0 1 1 0 1 0 1 0
0 0 1 1 1 1 1 1

Example 1.2.3. Find the (i) Converse, (ii) Contrapositive and (iii) Inverse of
the following statements:
If the function is differentiable, then it is integrable.

Solution

(i) If a function is integrable, then it is differentiable.

(ii) If a function is not integrable, then it is not differentiable.

(iii) If the function is not differentiable, then it is not integrable.

Definition 1.2.4. (Biconditional Statement): Let P and Q be any two


statements. The compound statement “P if and only if Q” is called the
biconditional statement of statements P and Q. Symbolically we denote this by
P ⇔ Q. The “if and only if” can be abbreviated as “iff”.

We can illustrate the truth table of P ⇔ Q as follows:

P Q P ⇔Q
1 1 1
1 0 0
0 1 0
0 0 1

9
1.3 Laws of Logic
Definition 1.3.1. (Logically Equivalent Statements): Two statements, P
and Q, are said to be logically equivalent if their truth values coincide. We
denote this by P ≡ Q.

Example 1.3.2. Show the following logically equivalent statements using truth
table.

(i) (P ⇒ Q) ≡ ¬P ∨ Q

(ii) ¬(P ∧ Q) ≡ ¬P ∨ ¬Q

(iii) ¬(P ∨ Q) ≡ ¬P ∧ ¬Q

(iv) ¬(P ⇒ Q) ≡ P ∧ ¬Q

Solutions

We will do (i) and the rest will be left as an exercise.

P Q ¬P P ⇒Q ¬P ∨ Q
1 1 0 1 1
1 0 0 0 0
0 1 1 1 1
0 0 1 1 1

As we can see the last two columns have the same truth values so the two
statements are logically equivalent.

Definition 1.3.3. (Tautology): Tautology is a statement that is always true.

Definition 1.3.4. (Contradiction): Contradiction is a statement that is al-


ways false.

Example 1.3.5. Determine whether the following statements are tautology,


contradiction or neither of the two.

(i) ((P ⇒ Q) ∧ ¬Q) ⇒ ¬P

10
(ii) ¬((P ∧ Q) ⇒ P )

(iii) P ⇔ (¬P ∧ (P ∨ Q))

Solution

(i) ((P ⇒ Q) ∧ ¬Q) ⇒ ¬P

P Q ¬P ¬Q P ⇒Q (P ⇒ Q) ∧ ¬Q ((P ⇒ Q) ∧ ¬Q) ⇒ ¬P
1 1 0 0 1 0 1
1 0 0 1 0 0 1
0 1 1 0 1 0 1
0 0 1 1 1 1 1

Last column shows the statement is always true, hence tautology.

(ii) ¬((P ∧ Q) ⇒ P )

P Q P ∧Q (P ∧ Q) ⇒ P ¬((P ∧ Q) ⇒ P )
1 1 1 1 0
1 0 0 1 0
0 1 0 1 0
0 0 0 1 0

Last column shows the statement is always false, hence contadiction.

(iii) P ⇔ (¬P ∧ (P ∨ Q))

P Q ¬P P ∨Q ¬P ∧ (P ∨ Q) P ⇔ (¬P ∧ (P ∨ Q))
1 1 0 1 0 0
1 0 0 1 0 0
0 1 1 1 1 0
0 0 1 0 0 1

From last column we can conclude the statement is neither tautology


nor contradiction.

11
Logically Equivalences
Equivalence Name
P ∧0≡0
P ∨1≡1 Domination law
P ∧1≡P
P ∨0≡P Identity law
P ∧P ≡P
P ∨P ≡P Idempotent law
¬(¬P ) ≡ P Double negation law
P ∧Q≡Q∧P
P ∨Q≡Q∨P Commutative laws
(P ∧ Q) ∧ R ≡ P ∧ (Q ∧ R)
(P ∨ Q) ∨ R ≡ P ∨ (Q ∨ R) Association laws
P ∧ (Q ∨ R) ≡ (P ∧ Q) ∨ (P ∧ R)
P ∨ (Q ∧ R) ≡ (P ∨ Q) ∧ (P ∨ R) Distributive laws
¬(P ∧ Q) ≡ ¬P ∨ ¬Q
¬(P ∨ Q) ≡ ¬P ∧ ¬Q De Morgan’s laws
P ∧ ¬P ≡ F
P ∨ ¬P ≡ T Negation law

Tutorials 1.3.6. Determine whether the following statements are tautology,


contradiction or neither of the two.

(i) (P ∨ Q) ⇒ P

(ii) (P ∧ (P ⇒ Q)) ⇒ Q

(iii) ((¬Q) ∧ (P ⇒ Q)) ⇒ (¬P )

(iv) (P ∨ Q) ⇒ (P ∧ Q)

(v) (Q ∧ (P ⇒ Q)) ⇒ P

(vi) (P ⇒ Q) ⇔ (Q ⇒ P )

12
1.4 Arguments
Definition 1.4.1. (Valid Argument):An argument is called a valid argu-
ment if the conjunction of hypotheses implies the conclusion. Otherwise the
argument is said to be invalid or also called fallacy.

We can prove the validity of the argument in two ways namely:

(a) By following the flow of the argument,

(b) by using truth tables.

Example 1.4.2. Investigate the validity of the following argument using

(a) Flow of the argument,

(b) by using truth tables.

13
(a)

P ⇒Q

R∨P

¬R

∴Q

(a) Flow of the argument,


If ¬R is true then R is false. If R is false and R ∨ P is true then P is true.
If P is true then for P ⇒ Q to be true then Q must be true. Then we
deduce that the argument is valid.

(b) by using truth tables,


we need to test whether the statement:

[(P ⇒ Q) ∧ (R ∨ P ) ∧ (¬R)] ⇒ Q

is tautology.
Let A ≡ (P ⇒ Q) ∧ (R ∨ P ) ∧ (¬R) in order to simplify our table.

P Q R ¬R P ⇒Q R∨P A ≡ (P ⇒ Q) ∧ (R ∨ P ) ∧ (¬R) A⇒Q


1 1 1 0 1 1 0 1
1 1 0 1 1 1 1 1
1 0 1 0 0 1 0 1
1 0 0 1 0 1 0 1
0 1 1 0 1 1 0 1
0 1 0 1 1 0 0 1
0 0 1 0 1 1 0 1
0 0 0 1 1 0 0 1

Example 1.4.3. Investigate the validity of the following argument using

(a) Flow of the argument,

14
(b) by using truth tables.

(a)

P ∨ ¬Q

R ⇒ Q

¬P

∴R

(a) Flow of the argument,


If ¬P is true then P is false. If P is false and P ∨ ¬Q is true then ¬Q is
true. If ¬Q is true, then Q is false. If Q is false and R ⇒ Q is true then R
is false. Since the conclusion is given R which in this instant we found it
to be false, then we deduce that the argument is invalid or it is a fallacy.

(b) by using truth tables,


we need to test whether the statement:

[(P ∨ ¬Q) ∧ (R ⇒ Q) ∧ (¬P )] ⇒ R

is tautology.
Let A ≡ (P ∨ ¬Q) ∧ (R ⇒ Q) ∧ (¬P ) in order to simplify our table.

P Q R ¬P ¬Q P ∨ ¬Q R⇒Q A ≡ (P ∨ ¬Q) ∧ (R ⇒ Q) ∧ (¬P ) A⇒R


1 1 1 0 0 1 1 0 1
1 1 0 0 0 1 1 0 1
1 0 1 0 1 1 0 0 1
1 0 0 0 1 1 1 0 1
0 1 1 1 0 0 1 0 1
0 1 0 1 0 0 1 0 1
0 0 1 1 1 1 0 0 1
0 0 0 1 1 1 1 1 0

15
Looking at the last column, we can clearly deduce the statement is not tautology,
hence the argument is not valid just as we expected.

Example 1.4.4. Investigate the validity of the following argument using

(a) Flow of the argument,

(b) by using truth tables.

(a) Mary plays netball or volleyball at school.


If Mary plays netball at school then she goes to Church.
Mary doesn’t go to Church.
∴ Mary plays volleyball.
Now let,
P : Mary plays netball at school.
Q : Mary plays volleyball at school.
R : Mary goes to church.

P ∨ Q

P ⇒ R

¬R

∴Q

(a) Flow of the argument,


If ¬R is true then R is false. If P ⇒ R is true and R is false then P is false.
If P ∨ Q is true and P is false then Q is true. Therefore the argument is
valid.

(b) We leave it as an exercise to students to verify using truth table that the
argument is valid by verifying the statement is tautology.

Example 1.4.5. Investigate the validity of the following argument using

(a) Flow of the argument,

16
(b) by using truth tables.

(a) If Mpho eats mangos then he lives in Venda.


Mpho either lives in Venda or Soweto.
Mpho does not live in Soweto.
∴ Mpho doesn’t eat mangos.
Now let,
P : Mpho eats mangos.
Q : Mpho lives in Venda.
R : Mpho lives in Soweto.

P ⇒ Q

Q ∨ R

¬R

∴ ¬P

(a) Flow of the argument,


If ¬R is true then R is false. If Q ∨ R is true and R is false then Q is true.
If P ⇒ Q is true and Q is true then P is either true or false hence 6 P is
also either true or false. So we cannot affirm that 6 P is true. Therefore
the argument is invalid.

(b) We again leave it as an exercise to students to verify using truth table


that the argument is invalid.

17
Tutorials 1.4.6. Test the validity of the following arguments using:

(a) Flow of the argument,

(b) by using truth tables.

1.

P ⇔ ¬R

Q ∨ R

¬P

∴Q

2.

P ⇒ Q

(Q ∨ R) ⇒ S

¬S

∴ ¬P

3.

P ⇒ ¬Q

Q ⇔ R

∴ ¬P

4.

S ∨ ¬R

P → R

P ∧ ¬Q

∴ ¬S

18
5. Pretty either works in mine or University.
Pretty eats KFC if and only if she doesn’t work in mine.
Pretty doesn’t work in University.
∴ Pretty doesn’t eat KFC..

6. Maggie went to the park or overseas.


If Maggie went to park or cinema then she eats burger and chips.
Maggie didn’t eat burger.
∴ Maggie went overseas..

7. If Mpho goes to school, she won’t play netball.


Mpho plays netball or drives a car if and only if she will be a teacher.
Mpho won’t be a teacher and she hate maths.
∴ Mpho doesn’t go to school.

Law of Syllogisms: Let P , Q and R be any two statements. If P ⇒ Q and


Q ⇒ R then P ⇒ R. This means the statement

((P ⇒ Q) ∧ (Q ⇒ R)) ⇒ (P ⇒ R).

is always true, i.e. observe that ((P ⇒ Q) ∧ (Q ⇒ R)) ⇒ (P ⇒ R) is tautology.


Chain Rule: If P1 , P2 , · · · , Pn are statements and P1 ⇒ P2 , P2 ⇒ P3 , · · · ,,
Pn−1 ⇒ Pn then P1 ⇒ Pn .

Example 1.4.7. If Finny cries then she is sick. If Finny is sick she won’t go to
school. If Finny doesn’t go to school she will fail the exam. If Finny fails the
exam she will be poor.
The above conditional statements imply the following statement: If Finny cries
then she will be poor.

19
1.5 Predicates
Definition 1.5.1. A predicate is a sentence which involves variables such that
if the variables are replaced by definite objects then a statement is obtained. We
will denote such sentences by: P (x), P (x, y), etc

Example 1.5.2. P (x) : 2 − x ≥ 5


P (x, y) : 3x + 5 = y.

Definition 1.5.3. (a) The symbol ∀ is called the universal quantifier and
represents the phrase “for all”, “for each”, or “for every”. The statement
(∀ ∈ U )(P (x)) is read “for all x ∈ U, P (x)”.

(b) The symbol ∃ is called the existential quantifier and represents the
phrase “there exists”, “there is”, or “for some”. The statement (∃ ∈
U )(P (x)) is read “there exists an x ∈ U , such that P (x)”.

Given a statement (∀x ∈ U )(P (x)) which is read “for all x in set A, then
P (x)”. U in this instance represent an unspecified set and P (x) is a statement
on x,
E.g: P (x) : 2 − x ≥ 5 and U can be set of real number (R), integers(Z),
natural numbers (N), etc.
In some instance our statements may involve more than one variable, set and
both two quantifiers.
E.g: (∀x ∈ A)(∃y ∈ B)(P (x, y)) which reads “for all x in set A, there exists a
y in B such that P (x, y).
It is of our interest to find out whether the given statement is true or false. Lets
look at some examples.

Example 1.5.4. Determine which of the following statements is true or false.


Justify your answer in each case.

(a) ∀x ∈ Z : −x ∈ N

(b) ∃x ∈ Z : −x ∈ N

20
(c) (∀x ∈ R)(∃y ∈ R : xy = 1

(d) (∃x ∈ R)(∀y ∈ R : xy = 1

(e) (∀x ∈ Q)(∃y ∈ R : x + y = 0

(f) (∃x ∈ Z)(∀y ∈ N : xy = 0

(g) (∃y ∈ R)(∀x ∈ R : x + xy = 8x

(h) (∀x ∈ R) : x2 > x

Solutions:

(a) ∀x ∈ Z : −x ∈ N it reads as:


“for every integer x, negative x is a natural number”.
Since we are interested in the set of all integers, we are interested in
finding out whether the negative of all integers is a natural number or
not. Such statement is false since for all positive integer x, −x will give
us negative integer which is not a natural number. For example, if x = 2,
−x = −2 ∈
/ N.

(b) ∃x ∈ Z : −x ∈ N it reads as:


“there exists an integer x, such that negative x is a natural number”.
The statement will be true since in this case we are only interested in
finding out if there is a particular integer which it’s negative is natural
number. There are infinitely many such x, actually we can only restrict
ourselves to negative integer, since their negative is positive integers which
are natural numbers.

(c) (∀x ∈ R)(∃y ∈ R : xy = 1


False, if x = 0, then xy = 0.

(d) (∃x ∈ R)(∀y ∈ R : xy = 1


False, each real has it’s own unique multiplicative inverse or in particular
if y = 0.

21
(e) (∀x ∈ Q)(∃y ∈ R : x + y = 0
True, every real x has its own additive inverse which is −x, i.e. y = −x.

(f) (∃x ∈ Z)(∀y ∈ N : xy = 0


True, when x = 0.

(g) (∃y ∈ R)(∀x ∈ R : x + xy = 8x


True, for y = 7.

(h) (∀x ∈ R) : x2 > x.


False, if x = 12 .

1.6 Negation of Quantified statements


Theorem 1.6.1. Let P (x) be an open statement in the variable x. Then
¬(∀x)(P (x)) is equivalent to (∃x)(¬P (x)) and ¬(∃x)(P (x)) is equivalent to
(∀x)(¬P (x)).

Example 1.6.2. Negate the following statements:

(a) (∀k ∈ Z)(∃r ∈ R)(r < k)


Negation will be

¬(∀k ∈ Z)(∃r ∈ R)(r < k) ≡ (∃k ∈ Z)(¬((∃r ∈ R)(r < k)))

≡ (∃k ∈ Z)(∀r ∈ R)(¬(r < k))

≡ (∃k ∈ Z)(∀r ∈ R)(r ≥ k)

(b) (∀ > 0)(∀n ∈ N)(∃m ∈ N)(|xn − xm | < )


Negation will be

¬[(∀ > 0)(∀n ∈ N)(∃m ∈ N)(|xn − xm | < )] ≡ (∃ > 0)¬[(∀n ∈ N)(∃m ∈ N)(|xn − xm | < )]

≡ (∃ > 0)(∃n ∈ N)¬[(∃m ∈ N)(|xn − xm | < )]

≡ (∃ > 0)(∃n ∈ N)(∀m ∈ N)¬[(|xn − xm | < )]

≡ (∃ > 0)(∃n ∈ N)(∀m ∈ N)(|xn − xm | ≥ )]

22
Tutorials 1.6.3. 1. Determine which of the following statements is true or
false. Justify your answer in each case.

(a) ∀x ∈ Z : 3x ∈ N

(b) ∃x ∈ N : x2 − x = 0

(c) (∀x ∈ R)(∃y ∈ Z : xy = 0)

(d) (∃x ∈ Z)(∀y ∈ Z : 7y − xy = 3y)

(e) (∀x ∈ Z)(∃y ∈ N : x + y = x)

(f) (∃x ∈ R)(∀y ∈ R : xy = x)

(g) (∀x ∈ R)(∃y ∈ R : xy = x)

(h) (∀ > 0)(∃n ∈ N : n > 1 )

2. Negate the following quantified statements.

(a) (∀ > 0)(∃δ > 0)(∀n ∈ N)(|xn − δ| ≤ )

(b) (∃x ∈ Z)(∃y ∈ R)(∀z ∈ R(x + y + z = 0)

(c) (∀x ∈ Z)(∀y ∈ Z)(∃z ∈ R(|x + y| > 0)

1.7 Methods of Proof


In this section we discuss some of the common methods of proof and the stan-
dard terminology that accompanies them. A mathematical system consists of
axioms, definitions, and undefined terms. An axiom is a statement that is
assumed true. A definition is used to create new concepts in terms of existing
ones. A theorem is a proposition that has been proved to be true. A lemma
is a theorem that is usually not interesting in it’s own right but is useful in
proving another theorem. A corollary is a theorem that follows quickly from
a theorem.

23
Methods of proofs to be discussed:

1. Direct proof

2. Indirect proof

• Proof by contradiction

• Proof by contrapositive

• Proof by counter-example.

3. Proof by cases.

Another important method of proof of great importance in mathematics is proof


by Mathematical Induction which we will reserve for the upcoming chapters.
The following definitions will be important in most of our discussions:

Definition 1.7.1. • An integer n is even if there exists an integer k such


that n = 2k. So the set of even integers is

E = {2k : k ∈ Z} = {. . . , −4, −2, 0, 2, 4, . . .}.

• An integer n is odd if there exists an integer k such that n = 2k + 1. So


the set of odd integers is

O = {2k + 1 : k ∈ Z} = {. . . , −3, −1, 1, 3, 5, . . .}.

1.7.1 Direct proof

In order to prove a conditional statement P ⇒ Q, we assume P and then deduce


Q by using some of axioms, definitions, rules or logical inferences, previously
proven theorems.

Example 1.7.2. Use the direct proof to prove that:


If n is an even integer then n2 is also an even integer.

Proof. In this statement we can let


P : n is even integer

24
Q : n2 is even integer.
We need to assume P and deduce Q.
Assume n is even, i.e. n = 2k for some k ∈ Z. Then

n2 =n·n

= (2k)(2k)

= 4k 2

= 2(2k 2 )

= 2m

for m = 2k 2 ∈ Z, hence n2 is even. 

Example 1.7.3. Use the direct proof to prove that


If n is even integer and m is odd integer, then n + m an odd integer.

Proof. In this statement we can let


P : n is even integer.
Q : m is odd integer.
R : n + m is odd integer.
We need to assume P ∧ Q and deduce R, i.e. P ∧ Q ⇒ R.
Assume n is even integer and m is an odd integer, i.e. n = 2k and m = 2r + 1
for some k, r ∈ Z. Then

n+m = (2k) + (2r + 1)

= 2(k + r) + 1

= 2q + 1

for q = k + r ∈ Z, hence n + m is odd integer. 

1.7.2 Indirect proof

Here we will look into three types of indirect proofs such as

25
• Proof by contradiction.

• Proof by contrapositive.

• Proof by counter-example.

Proof by Contradiction

In order to prove the statement P ⇒ Q using contradiction, we prove that the


statement is true by showing that it cannot possibly be false. That is, we assume
a statement is false and then arrive at some contradiction. It is easy to show
that
(P ⇒ Q) ≡ ¬P ∨ Q.

For us to prove using contradiction we assume (P ⇒ Q) is false i.e. ¬(P ⇒ Q)


is true. Also we can show that

¬(P ⇒ Q) ≡ P ∧ ¬Q

.
So in order to prove that (P ⇒ Q) using contradiction, we must assume P ∧ ¬Q
and therefore arrive at some contradiction.

Example 1.7.4. Prove by contradiction that if x ∈ R and x2 = 2, then x is


irrational.

Proof. We need to prove that P ⇒ Q where


P : x ∈ R and x2 = 2.
Q : x is irrational.
Assume P ∧ ¬Q i.e. Let x2 = 2 and assume that x is a rational number, i.e.
a
x= b where a, b ∈ Z with b 6= 0 and a and b do not have common divisor.
a2 a2
x2 = ( ab )2 = b2 . Since x2 = 2, then b2 = 2 and then a2 = 2b2 , hence a2 is an
even number. The only way this can happen is if a itself is even integer, i.e.
a = 2k for some k ∈ Z.
Then a2 = (2k)2 = 4k 2 . So 4k 2 = 2b2 , hence 2k 2 = b2 . This implies that b2 is
even hence b is also even, i.e. b = 2r for some r ∈ Z. But this contradicts a and

26
b not having a common divisor since in this case we have both a and b divisible
by 2. So our assumption was false, hence x is irrational number. 

Proof by contrapositive

To prove the statement P ⇒ Q by contrapositive, we must assume ¬Q and


deduce ¬P , hence ¬Q ⇒ ¬P .

Example 1.7.5. Given that n is an integer. Prove by contrapositive that if n2


is even, then n is even.

Proof. In this case we must show that ¬Q ⇒ ¬P where


P : n2 is an even integer.
Q : n is an even integer.
Suppose n is odd. Then n = 2k + 1 for some k ∈ Z. Then
n2 = (2k+1)2 = 4k 2 +4k+1 = 2(2k 2 +2k)+1 = 2m+1 where m = 2k 2 +2k ∈ Z,
which then leads to n2 being an odd integer. Then by contrapositive if n2 is
even, then n is even. 

Proof by Counter-Example

It is common to prove a false statement by using counter-example.

Example 1.7.6. Prove using counter-example that the following statement is


not correct:
For a, b integers, if a divides b and b divides a, then a = b.

Proof. Let a = 1 and b = −1. Clearly a, b ∈ Z and

a 1 −1 b
= = −1 = = .
b −1 1 a

In this case we have a divides b and b divides a but a 6= b. So our statement is


false by the above counterexample. 

27
1.7.3 Proof by cases

This type of proving involve us subdividing our proof into different cases.

Example 1.7.7. Prove by cases that if n is an integer, the n2 + n is an even


integer.

Proof. We will prove this by cases by first assuming n is even and another case
where we assume n is odd.

Case 1:

Assume n is even integer. Hence n = 2k for some k ∈ Z. then

n2 + n = (2k)2 + 2k = 4k 2 + 2k = 2(2k 2 + k) = 2r,

where r = 2k 2 + k ∈ Z since k ∈ Z. Therefore n2 + n is an even integer.

Case 2:

Assume n is odd integer. Hence n = 2k + 1 for some k ∈ Z. then

n2 +n = (2k+1)2 +2k+1 = 4k 2 +4k+1+2k+1 = 4k 2 +6k+2 = 2(2k 2 +3k+1) = 2p,

where p = 2k 2 + 3k + 1 ∈ Z since k ∈ Z. Therefore n2 + n is an even integer.


We can conclude that which ever integer n is, i.e. either even or odd, n2 + n is
an even integer. 

Tutorials 1.7.8. Prove the following using (i) Direct proof, (ii) Proof by
contradiction, (iii) Proof by contrapositive

1. Prove the following using Direct proof

(a) If n is odd integer then n2 + 2n is odd.

(b) If n is even integer then n2 + 1 is odd.

(c) If n is odd integer and m is odd integer then nm is odd.

(d) If n is odd integer and m is even integer then nm is even.

28
(e) If n is even integer and m is odd integer then n2 + m2 is odd.

2. Prove the following using proof by contradiction

(a) If n2 + 1 is odd then n is even.

(b) If m + n is even, then m and n are either both odd or both even.

(c) If mn is odd integer then both m and n are odd.

(d) If m is even integer then m + 1 is odd.

3. Prove the following using proof by contraposition

(a) If m is odd integer then m + 1 is even.

(b) If m + n is even, then m and n are either both odd or both even.

(c) If n2 + 1 is odd then n is even integer.

4. Use proof by cases to prove that

(a) For every integer n, n3 + n is even.

(b) For any a, b ∈ R, |ab| = |a| · |b|.

(c) For any n ∈ N, n(n + 1) is even.

“Do not worry about your difficulties in Mathematics. I can assure you mine
are still greater.” Albert Einstein

29
Chapter 2

SET THEORY AND


RELATION

“Mathematics is the most beautiful and most powerful creation of the human
spirit”. Stefan Banach

2.1 Sets
Sets play a major role in the foundation of Mathematics. Sets are used to
group objects together. Sets may or may not satisfy a certain property. For
instance, we can take a collection of all students enrolled in the University of
Limpopo for year 2017 to form a set. Furthermore we can also describe other sets
within a particular bigger set like for instance taking a set of students in faculty
of Science and Agriculture, School of Mathematical and Computer Sciences,
Students studying SMTB000, etc. It is possible that certain individuals (or
objects) may appear in more than one set.

Definition 2.1.1. A set is an unordered collection of well defined objects called


the elements or members of the set.

“Member of” or “element of” is a relation between objects and sets. If a is an

30
object and A is a set, then a ∈ A means a is a member of A or ‘a is an element
of A’. Then a ∈
/ A means a is not a member of A or ‘a is not an element of A’

2.1.1 Listing of elements

There are two methods of representing a set which are:

1. Listing the elements in a roster: Here we list elements between the


two curly brackets (braces).
e.g. The notation {a, 1, e, 7} represents the set with four elements a, 1, e,
and 7. We call this a roster method.
Sometimes the roster method can be used to describe a set without listing
all the members. This is done by listing some of the members, and then
ellipses (. . .) are used when the general pattern of the elements is obvious.
e.g. D = {1, 4, 9, 16, . . . , 81, 100} which is clearly a set of all perfect square
numbers from 1 to 100. One can easily make note of obvious elements not
listed, for example, 36 ∈ D and 40 ∈
/ D.

2. Set builder notation: We characterized the elements of that set by a


specific property or properties they must have in order to be a member of
such set.
e.g. A = {x : x is a prime number between 6 and 15}.
We often use this method when it is impossible to list such elements,
though in some instances it can be easy to list such element. From our ex-
ample it easy to list elements in set A, i.e. A = {7, 11, 13}; but it is not pos-
sible to list elements of set B = {x : x = ab , where a and b are negative integers},
since there are infinite such number in set B.

2.1.2 Important sets of numbers:

1. Natural numbers: Natural numbers are also known as counting num-


bers, beginning at 1. The set of natural numbers is denoted by N; i.e.

N = {1, 2, 3, 4, . . .}.

31
N is an infinite set.
If we include a zero we obtain the set of whole numbers denoted by N0 .
So
N0 = {0, 1, 2, 3, 4, . . .}

2. Integers: Integers consist of natural numbers, negative natural numbers


and a zero. The set if integers is denoted by Z; i.e.

Z = {. . . , −3, −2, −1, 0, 1, 2, 3, . . .}.

• Positive integers are integers greater than zero and represented as


follows;
Z+ = {1, 2, 3, 4, . . .}.

• Negative integers are integers less than zero and represented as


follows;
Z− = {−1, −2, −3, −4, . . .}.

a
3. Rational numbers are numbers that can be written in the form b where
a, b ∈ Z and b 6= 0; i.e.
a
Q={ : a, b ∈ Z and b 6= 0}.
b

Note that all integers are rational numbers with b = 1.

4. Irrational numbers are the opposite of rational, i.e. those numbers that
a
cannot be written as b where a, b ∈ Z and b 6= 0. These can be represented
as R \ Q or R − Q or Qc (complement of Q), where R is the set of real
numbers to be discussed next.

5. Real numbers consist of all the numbers in the sets discussed above, i.e.
more specifically by combining the rational and irrational numbers we get
the real numbers.

6. Complex numbers are numbers of the form a + ib where a, b ∈ R and


i2 = −1. (See Chapter 6)

32
Definition 2.1.2. A Universal set ia a set which contains all sets including
itself. We usually denote universal set by U .

We will assume all sets come from universal set.

2.1.3 Subsets

The set A is a subset of B if and only if every element of A is also an element


of B. We use the notation A ⊆ B to indicate that A is a subset of B.
To show that A ⊆ B, we must show that if x ∈ A, then also x ∈ B.
E.g. If A = {1, 2, 3, a, b, c} and B = {2, 3, b}, then B ⊆ A, since every element
in B is in A.
The statement A ⊆ B means that A may equal B, otherwise if A 6= B, we may
write A ⊂ B and we say A is a proper subset of B. From above example it
should be clear that B ⊂ A.

2.1.4 Equal sets

Two sets are equal if they have the same element, and we write A = B if set A
and B are equal sets.
To show that two sets are equal, we must show that A ⊆ B and B ⊆ A. Note
that if A = {1, a, 7} and B = {7, 1, a}, then A = B since both sets contain the
same elements. So the order in which the elements of a set are listed does not
matter. Also set {1, 1, 1, a, a, 7, 7, 7} and {1, a, 7} are equal since both contain
same elements, so the repeat elements are just taken as one element.

2.1.5 The empty set

An empty set is a set that contains no elements. It is also called the null set
and is denoted by ∅ or {}.
Singleton is a set that contains one element. Note that ∅ =
6 {∅}.The set {∅} is
a singleton since it is a set containing one element ∅.

33
Definition 2.1.3. Cardinality of a set: The Cardinality of set represent the
number of elements in a given set. If set A has n elements, we say that A is a
finite set and n is the cardinality of A. The cardinality of A is denoted by |A|.
E.g. If A = {1, 2, 3, a, b, c}, then |A| = 6.
If X is a set of all letters in English alphabet, then |X| = 26.

2.1.6 Set Operations

Two or more sets may be combined in many different ways. Here we will discuss
the different ways of combining sets.

Definition 2.1.4. Let A and B be sets. The intersection of A and B denoted


by A ∩ B, is the set containing those elements in both A and B; i.e.

A ∩ B = {x : x ∈ A and x ∈ B} = {x : (x ∈ A) ∧ (x ∈ B)}.

Definition 2.1.5. Let A and B be sets. The union of A and B denoted by


A ∪ B, is the set containing those elements that are in either A or B, or in both;
i.e.
A ∪ B = {x : x ∈ A or x ∈ B} = {x : (x ∈ A) ∨ (x ∈ B)}.

Definition 2.1.6. The difference or relative complement between sets A


and B A \ B = A − B is the set of elements of A that are not in B. i.e.

A \ B = A − B = {x : x ∈ A and x ∈
/ B} = {x : (x ∈ A) ∧ (x ∈
/ B)}.

Note that B \ A = B − A = {x : x ∈ B and x ∈


/ A} and A \ B = A − B = {x :
x ∈ A and x ∈
/ B} are clearly not the same sets.

Note that A \ B = A ∩ B 0

Example 2.1.7. Let A = {1, 3, 5, a, b, d, f }, B = {3, 4, 5, b, c, g} and C =


{2, e, f }.
Intersection: A ∩ B = {3, 5, b}; A ∩ C = {f }; B ∩ C = ∅; A ∩ B ∩ C = ∅.
Union: A∪B = {1, 3, 4, 5, a, b, c, d, f, g}; A∪C = {1, 2, 3, 5, a, b, d, e, f }; B ∪C =
{2, 3, 4, 5, b, c, e, f, g}; A ∪ B ∪ C = {1, 2, 3, 4, 5, a, b, c, d, e, f, g}.
Difference: A − B = {1, a, d, f }; C − A = {2, e}; check the rest.

34
19
Example 2.1.8. If A = {x : x is positive odd numbers less than or equal to 2 }

and B = {x ∈ Z : 0 ≤ x2 < 18}, then find:


(a) A ∩ B, (b) A ∪ B and (c) B − A.
Solution:
The first thing to do is to list the elements of set A and B so that we can easily
see which elements are common or not in both sets.
So A = {1, 3, 5, 7, 9} and B = {−4, −3, −2, −1, 0, 1, 2, 3, 4}.

(a) A ∩ B = {1, 3}.

(b) A ∪ B = {−4, −3, −2, −1, 0, 1, 2, 3, 4, 5, 7, 9}.

(c) B − A = {−4, −3, −2, −1, 0, 2, 4}.

Tutorials 2.1.9. 1. Given the following sets:


A = {x ∈ N : x2 ≤ 50}, B = {x ∈ Z : − 37 ≤ x < 4},
C = {x ∈ R : |x| < 2}, and
D = {x : x is prime numbers less than or equal to 30}.
Find

(i) A ∩ B

(ii) A ∪ B

(iii) A ∩ C

(iv) B ∩ D

(v) A ∪ D

(vi) (A − C) ∪ (D − B)

(vii) B − C

(viii) (A ∩ B) ∪ (B ∩ C)

(ix) C − B (challange)

2. Determine the cardinality of your answers from (i) to (ix) above.

35
2.2 Set Algebra
Theorem 2.2.1. Let A; B; C be sets. Then

(a) A ∪ ∅ = A

(b) A ∪ A = A

(c) A ∪ B = B ∪ A

(d) (A ∪ B) ∪ C = A ∪ (B ∪ C).

Proof. We’ll prove (a) and (d).


(d) Remember that we are required to show both inclusions.
Now, let x ∈ (A ∪ B) ∪ C ⇒ x ∈ (A ∪ B) or x ∈ C.
If x ∈ C then x ∈ B ∪ C, and so x ∈ A ∪ (B ∪ C).
On the other hand, if x ∈ A ∪ B, then either x ∈ A ⇒ x ∈ A ∪ (B ∪ C) or
x ∈ B ⇒ x ∈ B ∪ C hence x ∈ A ∪ (B ∪ C).
So in all cases x ∈ A ∪ (B ∪ C).
Hence (A ∪ B) ∪ C ⊆ A ∪ (B ∪ C).
We now prove the other inclusion.
Let x ∈ A ∪ (B ∪ C) ⇒ x ∈ A or x ∈ B ∪ C.
If x ∈ A then x ∈ A ∪ B, hence x ∈ (A ∪ B) ∪ C.
If x ∈ B ∪ C, then either x ∈ B ⇒ x ∈ (A ∪ B) ∪ C or x ∈ C ⇒ x ∈ (A ∪ B) ∪ C.
So in all cases x ∈ (A ∪ B) ∪ C. Hence A ∪ (B ∪ C) ⊆ (A ∪ B) ∪ C.
So in conclusion A ∪ (B ∪ C) = (A ∪ B) ∪ C.

(b) and (c) is left as an exercise. 

Theorem 2.2.2. Let A; B; C be sets. Then

(a) A ∩ ∅ = ∅

(b) A ∩ A = A

(c) A ∩ B = B ∩ A

36
(d) (A ∩ B) ∩ C = A ∩ (B ∩ C).

Proof: Exercise

Theorem 2.2.3. If A, B, C be sets, then

(a) A ∩ (B ∪ C) = (A ∩ B) ∪ (A ∩ C)

(b) A ∪ (B ∩ C) = (A ∪ B) ∩ (A ∪ C).

Proof. (a)

Let x ∈ A ∩ (B ∪ C)

⇒ x ∈ A and x ∈ B ∪ C

⇒ x ∈ A and (x ∈ B or x ∈ C)

⇒ (x ∈ A and x ∈ B) or (x ∈ A and x ∈ C)

⇒ x ∈ A ∩ B or x ∈ A ∩ C.

i.e. x ∈ (A ∩ B) ∪ (A ∩ C). Thus A ∩ (B ∪ C) ⊆ (A ∩ B) ∪ (A C).


Conversely, let x ∈ (A ∩ B) ∪ (A ∩ C).
If x ∈ A ∩ B then x ∈ A and x ∈ B ⇒ x ∈ A and x ∈ B ∪ C
so x ∈ A ∩ (B ∪ C).
If x ∈ A ∩ C then x ∈ A and x ∈ C ⇒ x ∈ A and x ∈ B ∪ C.
so x ∈ A ∩ (B ∪ C).
Hence (A ∪ B) ∩ (A ∪ C) ⊆ A ∪ (B ∩ C).
Hence (A ∪ B) ∩ (A ∪ C) = A ∪ (B ∩ C).

(b) Let x ∈ A ∪ (B ∩ C) ⇒ x ∈ A or x ∈ B ∩ C.
If x ∈ A then x ∈ A ∪ B and x ∈ A ∪ C.
So x ∈ (A ∪ B) ∩ (A ∪ C).
If instead x ∈ B ∩ C then x ∈ B and x ∈ C. So x ∈ A ∪ B and x ∈ A ∪ C,
i.e. x ∈ (A ∪ B) ∩ (A ∪ C).
Hence A ∪ (B ∩ C) ⊆ (A ∪ B) ∩ (A ∪ C).

37
Conversely, if y ∈ (A ∪ B) ∩ (A ∪ C) ⇒ y ∈ A ∪ B and y ∈ A ∪ C,
i.e. y is in both unions. ⇒ (y ∈ A or y ∈ B) and (y ∈ A or y ∈ C). So
either y ∈ A or y ∈
/ A.
case 1: y ∈ A ⇒ y ∈ A ∪ (B ∩ C),
Otherwise
case 2: y ∈ B ∩ C so A ∪ (B ∩ C) which proves that (A ∪ B) ∩ (A ∪ C) ⊆
A ∪ (B ∩ C).


Proposition 2.2.4. If A and B are sets, then the following are equivalent
(TFAE):

(a) A ⊆ B

(b) A ∩ B = A

(c) A ∪ B = B.

Proof. (a) ⇒ (b) :


Assume A ⊆ B, We will have to show that A ∩ B ⊆ A and A ⊆ A ∩ B.
Now A ⊆ B ⇒ ∀x ∈ A, x ∈ B, then x ∈ A ∩ B, hence A ⊆ A ∩ B.
But A ∩ B ⊆ A always.
So A = A ∩ B.

(b) ⇒ (a) :
Assume A ∩ B = A. We will show that A ⊆ B. Now Since A ∩ B = A
⇒ ∀x ∈ A, x ∈ A ∩ B ⇒ x ∈ B. So A ⊆ B.

(b) ⇒ (c):
Assume A ∩ B = A. We must show that A ∪ B ⊆ B and B ⊆ A ∩ B.
Let x ∈ A ∪ B. Then x ∈ A or x ∈ B.
If x ∈ A, then x ∈ A ∩ B by hypothesis, which also imply that x ∈ B.
So either case we end up with x ∈ A ∪ B ⇒ x ∈ B, hence A ∪ B ⊆ B.

38
Also B ⊆ A ∪ B always.

(c) ⇒ (b) :
Assume A ∪ B = B. We show A ∩ B ⊆ A and A ⊆ A ∩ B.
Now, ∀x ∈ A, x ∈ B since A ∪ B = B ⇒ x ∈ A ∩ B, hence A ⊆ A ∩ B.
Now since A ∩ B ⊆ A always, we conclude that A = A ∩ B.

(a) ⇒ (c) : Assume A ⊆ B. We show A ∪ B ⊆ B and B ⊆ A ∪ B.


Let x ∈ A ∪ B ⇒ x ∈ A or x ∈ B.
If x ∈ A then x ∈ B since A ⊆ B, so in either case x ∈ A ∪ B ⇒ x ∈ B.
So A ∪ B ⊆ B.
Also B ⊆ A ∪ B always. Hence A ∪ B = B.

(c) ⇒ (a) : Assume A ∪ B = B. We show A ⊆ B. Now A ⊆ A ∪ B always.


⇒ A ⊆ B since A ∪ B = B. 

Definition 2.2.5. Let A be a set. Then the set of elements that do not belong
to the set A is called the complement of A, and this set is denoted A0 .

Proposition 2.2.6. Let A, B be sets. Then

(a) (A0 )0 = A

(b) If A ⊆ B then B 0 ⊆ A0

(c) (A ∪ B)0 = A0 ∩ B 0

(d) (A ∩ B)0 = A0 ∪ B 0 .

Proof. We’ll prove (a) and (c) and then (b) and (d) will be left for exercise.
(a) We must show that (A0 )0 ⊆ A and A ⊆ (A0 )0 .

Let x ∈ (A0 )0 ⇔ x ∈
/ A0

⇔ x ∈ A.

39
⇒ (A0 )0 ⊆ A and A ⊆ (A0 )0 , hence (A0 )0 = A

(c)Here we prove that (A ∪ B)0 ⊆ A0 ∩ B 0 and A0 ∩ B 0 ⊆ (A ∪ B)0 .

Let x ∈ (A ∪ B)0

⇔x∈
/ (A ∪ B)

⇔x∈
/ A and x ∈
/B

⇔ x ∈ A0 and x ∈ B 0

⇔ x ∈ A0 ∩ B 0

⇒ (A ∪ B)0 ⊆ A0 ∩ B 0 and A0 ∩ B 0 ⊆ (A ∪ B)0 , hence (A ∪ B)0 = A0 ∩ B 0 . 

Tutorials 2.2.7. 1. Let A, B and C be sets. Prove that

(i) (A − B) − C = (A − C) − (B − C) Use the fact that A − B = A ∩ B 0 .

(ii) If A ⊆ C and B ⊆ C then A ∪ B ⊆ C.

2. Let A and B be sets. Prove that

(i) A ∩ B = ∅ ⇔ A ⊆ B 0 .

(ii) If A ∩ B = A ∪ B ⇔ A = B.

(iii) If B ⊆ C then A ∪ B ⊆ A ∪ C for any set A.

2.2.1 Cartesian Products

Definition 2.2.8. Given two sets A and B, the cartesian product of A and B,
denoted by A × B, is the set of all ordered pairs (a, b), where a ∈ A and b ∈ B,
hence
A × B = {(a, b) : a ∈ A and b ∈ B}.

Example 2.2.9. If A = {3; 4} and B = {5, 7, 8}, then

A × B = {(3, 5), (3, 7), (3, 8), (4, 5), (4, 7), (4, 8)}.

40
and
B × A = {(5, 3), (5, 4), (7, 3), (7, 4), (8, 3), (8, 4)}.

Notice that cartesian product is in general not commutative i.e. A × B 6= B × A.


From our example, we see that (3, 5) ∈ A × B, but (3, 5) ∈
/ B × A. Note that
(x, y) = (x0 , y 0 ) ⇒ x = x0 and y = y 0 . So take note of the fact (x, y) is different
from {x, y}. If A, B are finite sets, then cardinality |A × B| = |A| · |B| (verify!).
Note! (A × B) × C 6= A × (B × C). Why not? What does equality of sets
mean? Observe! ((x, y), z) ∈ (A × B) × C and (x, (y, z)) ∈ A × (B × C) for all
x ∈ A, y ∈ B, z ∈ C.

Proposition 2.2.10. If A, B, C are sets, then

(a) (A ∪ B) × C = (A × C) ∪ (B × C)

(b) (A ∩ B) × C = (A × C) ∩ (B × C)

Proof. (a) Let

(x, y) ∈ (A ∪ B) × C

⇒ x ∈ A ∪ B and y ∈ C

⇒ x ∈ A or x ∈ B and y ∈ C. ⇒ (x, y) ∈ (A × C) or (x, y) ∈ (B × C).

So (A ∪ B) × C ⊆ (A × C) ∪ (B × C).
Conversely, let

(p, q) ∈ (A × C) ∪ (B × C)

⇒ (p, q) ∈ A × C or (p, q) ∈ B × C

⇒ p ∈ A and q ∈ C or p ∈ B and q ∈ C

⇒ p ∈ A ∪ B and q ∈ C, i.e. (p, q) ∈ (A ∪ B) × C.

So (A × C) ∪ (B × C) ⊆ (A ∪ B) × C.
Hence (A × C) ∪ (B × C) = (A ∪ B) × C. (b) Exercise! 

Theorem 2.2.11. Let A, B, C, D be sets. Then

41
(a) (A × B) ∩ (C × D) = (A ∩ C) × (B ∩ D)

(b) (A × B) ∪ (C × D) ⊆ (A ∪ C) × (B ∪ D)

Proof. (a) Let (x, y) be an ordered pair. Then

(x, y) ∈ (A × B) ∩ (C × D)

⇔ (x, y) ∈ A × B and (x, y) ∈ C × D

⇔ (x ∈ A and y ∈ B) and(x ∈ C and y ∈ D)

⇔ x ∈ A ∩ C and y ∈ B ∩ D

⇔ (x, y) ∈ (A ∩ C) × (B ∩ D).

So (A × B) ∩ (C × D) = (A ∩ C) × (B ∩ D)
(b) Let (x, y) ∈ (A × B) ∪ (C × D). Then

(x, y) ∈ A × B OR (x, y) ∈ C × D

⇒ (x ∈ A and y ∈ B) OR (x ∈ C and y ∈ D)

⇒ (x ∈ A or x ∈ C) and (y ∈ B or y ∈ D)

⇒ x ∈ A ∪ C and y ∈ B ∪ D

⇒ (x, y) ∈ (A ∪ C) × (B ∪ D)

⇒ (A × B) ∪ (C × D) ⊆ (A ∪ C) × (B ∪ D).

The notion of an ordered pair generalizes to what is known as an ordered n-


tuple. The n-tuple of n objects a1 , a2 , . . . an ; an (given in that order) is an object
(a1 , a2 , . . . , an ) having the property that (a1 , a2 , . . . , an ) = ((b1 , b2 , . . . , bn ) iff
a1 = b1 ; a2 = b2 ; . . . ; an = bn . For example (1, 2, 3, 4, 5), (5, 4, 3, 2, 1), (5, 4, 3, 1, 2)
are all different 5-tuple.

Tutorials 2.2.12. 1. Let A, B, C and D be sets. Prove that

(i) A × (B − C) = (A × B) − (A × C)

(ii) If A × C = B × C, and C 6= ∅, then A = B (challange).

42
2. Use Question 1 in Tutorial 2.1.9 to find:
(i) A × B; (ii) (A × C) ∪ (B × C) and
(iii)also verify that (A ∩ B) × C = (A × C) ∩ (B × C).
For set C use C = {x ∈ Z : |x| < 2} instead.

3. Find cardinality of sets sets from (i) to (iii) in Question 2. above.

2.2.2 Power Set

Consider set A = {a, b, 3}. The list of all subsets of A is:

∅, {a}, {b}, {3}, {a, b}, {a, 3}, {b, 3}, {a, b, 3}.

The set which contains all these 8 elements is called the power set of A.

Definition 2.2.13. The power set P(A) of a set A is the collection of all subsets
of A.i.e. P(A) = {X : X ⊆ A} = set of all subsets of A.

Thus, for any set X, the statements X ⊆ A and X ∈ P(A) are equivalent.

Example 2.2.14. (a) P({3}) = {∅, {3}}

(b) Let A = {∅, {2}}. Then

P(A) = {∅, {∅}, {{2}}, {∅, {2}}}.

Proposition 2.2.15. P(∅) = {∅}.

Proof. We must show that P(∅) ⊆ {∅} and {∅} ⊆ P(∅}. If A ∈ P(∅) ⇒ A ⊆
∅ ⇒ A = ∅. For if A 6= ∅; then A has a member x and the inclusion A ⊆ ∅, would
force x ∈ ∅ which is absurd since ∅ is an empty set. Thus we have A ∈ {∅}.
Hence P(∅) ⊆ {∅}.
Conversely, ∅ ⊆ ∅ (since any set is a subset of itself). Hence ∅ ∈ P(∅), and
therefore {∅} ⊆ P(A). consequently P(∅) = {∅}. 

43
Observation: If A has n elements, then P(A) has 2n elements, i.e.

|P(A)}| = 2n .

In other words, a set with n elements has 2n subsets. This can be proved by
induction. We however omit the proof here since induction is the subject of
another chapter.

Lemma 2.2.16. (a) If X ⊆ Z and Y ⊆ Z then X ∪ Y ⊆ Z.

(b) If Z ⊆ X and Z ⊆ Y then Z ⊆ X ∩ Y .

Proof. (a) If x ∈ X ∪ Y then either x ∈ X or x ∈ Y . In either case x ∈ Z


since X ⊆ Z and Y ⊆ Z. Therefore X ∪ Y ⊆ Z.

(b) If x ∈ Z then x ∈ X and x ∈ Y since Z ⊆ X and Z ⊆ Y . Thus x ∈ X ∩ Y


and so Z ⊆ X ∩ Y .


Theorem 2.2.17. Let A and B be sets. Then

(a) {∅, A} ⊆ P(A)

(b) A ⊆ B ⇔ P(A) ⊆ P(B)

(c) P(A) ∪ P(B) ⊆ P(A ∪ B)

(d) P(A) ∩ P(B) = P(A ∩ B).

Proof. (a) We already know that ∅ ⊆ A and A ⊆ A ⇒ ∅ ∈ P(A) and A ∈


P(A). Hence {∅, A} ⊆ P(A).

(b) (⇒) : Assume A ⊆ B, and let X ∈ P(A). We want to show that X ∈ P(B)
which is equivalent to showing that X ⊆ B.
Now, X ∈ P(A) ⇒ X ⊆ A ⊆ B by hypothesis. So X ⊆ B, that is
X ∈ P(B). Hence P(A) ⊆ P(B).
(⇐): Conversely, assume P(A) ⊆ P(B). We have x ∈ A ⇒ {x} ⊆ A ⇒
{x} ∈ P(A) ⇒ {x} ∈ P(B), since P(A) ⊆ P(B) ⇒ {x} ⊆ B ⇒ x ∈ B,
and thus A ⊆ B. The result is therefore proved.

44
(c) Since A ⊆ A ∪ B and B ⊆ A ∪ B, then by part (b) we have that P(A) ⊆
P(A ∪ B) and P(B) ⊆ P(A ∪ B), and by lemma (1.1 2.2.16 we have
P(A) ∪ P(B) ⊆ P(A ∪ B).
OR
Let X ∈ P(A) ∪ P(B), then X ∈ P(A) or X ∈ P(B). This implies that
X ⊆ A or X ⊆ B, hence X ⊆ A ∪ B. So X ∈ P(A ∪ B), hence our result.

(d) (⇒) Since A ∩ B ⊆ A, we have P(A ∩ B) ⊆ P(A) by part (b). Similarly


P(A ∩ B) ⊆ P(B). So lemma (2.2.16 (b)) gives P(A ∩ B) ⊆ P(A) ∩ P(B).

(⇐) : Conversely, if X ∈ P(A) ∩ P(B) then X ∈ P(A) and X ∈ P(B),


⇒ X ⊆ A and X ⊆ B. So X ⊆ A ∩ B ⇒ X ∈ P(A ∩ B). Therefore
P(A) ∩ P(B) ⊆ P(A ∩ B) and we have proved the desired equality.


Tutorials 2.2.18. 1. Find power sets of the following sets:


A = {0, b}; B = {{7}}; C = {∅, {3}, d} and D = P({5}).

2. Consider sets A = {x ∈ N : −3 ≤ x < 27 }, B = {x ∈ R : x is prime number less than 25},


Find P(A ∩ B) and P(A − B) and hence determine their cardinality as
well.

2.2.3 Symmetric difference

Definition 2.2.19. We define symmetric difference to be

A ⊕ B = (A − B) ∪ (B − A).

Prove that

(i) A ⊕ A = ∅

(ii) A ⊕ ∅ = A

(iii) (A ⊕ B) − (A − B) = B − A

(iv) A ⊕ B = ∅ ⇒ A = B

45
2.3 Relations
Definition 2.3.1. A relation R from a set A to a set B is simply any subset
of the cartesian product A × B, i.e. a collection of ordered pairs (x, y) : x ∈ A
and y ∈ B.

Example 2.3.2. Let A = {1, 3}, B = {2, 3, 4}. Then we can define the following
relations from A to B:

1. R1 = {(1, 4)}

2. R2 = {(1, 2), (1, 3), (3, 4)}

3. R3 = {(1, 2), (3, 3)}

4. R4 = {(1, 2), (1, 3), (3, 2), (3, 3), (3, 4)}

Observe that in each case Ri ⊆ A × B, i = 1, 2, 3, 4.

Definition 2.3.3. Suppose R is a relation from A to B. Then for (a, b) ∈ R


we say that a relates to b under R or a is related to b under R written aRb.

Example 2.3.4. Let A = {1, 2, 3, 4}.Define relation R = {(a, b) : a divides b}.


Solution:

R = {(1, 1), (1, 2), (1, 3), (1, 4), (2, 2), (2, 4), (3, 3), (4, 4)}.

Example 2.3.5. Let A = R and B = R. We define the relation on R from R


to R by aRb iff a2 − b2 = 1, i.e.

R = {(a, b) : a2 − b2 = 1}.

The set R contains infinitely many elements, e.g. (2, 3), (1, 0),etc.

2.3.1 Types of Relations

There are several properties that are used to classify relations on a set. We will
introduce the most important of these here. Let R be a relation from a set A
to the set A itself so that R ⊆ A × A. Then

46
1. R is called a reflexive relation if for all a ∈ A, (a, a) ∈ R, i.e. aRa.

2. The relation R from A to A is called a symmetric relation if whenever


(a, b) ∈ R, (b, a) ∈ R and a, b ∈ A, i.e. aRb ⇒ bRa.

3. R is called an antisymmetric relation if for any a, b ∈ A, (a, b) ∈ R and


(b, a) ∈ R means that a = b, i.e. aRb and bRaa = b.

4. The relation R from A to A is called a transitive relation if for any


a, b, c ∈ A, (a, b) ∈ R, (b, c) ∈ R ⇒ (a, c) ∈ R, i.e. aRb and bRc ⇒ aRc.

Example 2.3.6. The relation R from Z to Z defined by xRy iff x = y:

• R is reflexive since for any x ∈ Z, x = x.

• If xRy ⇒ x = y hence since equality has commutative property in Z,


y = x ⇒ yRx, so R is symmetric.

• Since xRy and yRx ⇒ x = y, then R is antisymmentric.

• Let xRy and yRz; then x = y and y = z. By equality property, x = z as


well. So R is transitive.

Example 2.3.7. The relation R from R to R defined by xRy iff x ≥ y:

• R is reflexive since for any x ∈ R, x = x, i.e. x ≥ x..

• If xRy ⇒ x ≥ y, then −y ≥ −x ⇒ y ≤ x, hence not y ≥ x. So R is not


symmetric.

• Since xRy ⇒ x ≥ y and yRx ⇒ y ≥ y, then x = y, hence R is antisym-


mentric.

• Let xRy and yRz; then x ≥ y and y ≥ z.Then x ≥ z as well. So R is


transitive.

Definition 2.3.8. A relation on a set A is called an equivalence relation if


it is reflexive, symmetric, and transitive.

47
Example 2.3.9. Let A = {a, b, c}. Then we can define the following relations
from A to B:

1. R1 = {(a, a), (a, b), (b, c)}

2. R2 = {(a, b), (b, a), (a, c), (c, a)}

3. R3 = {(a, a), (b, b), (a, b), (a, c), (b, c), (b, a)}

4. R4 = {(a, a), (a, b), (a, c), (b, c), (c, b), (b, b), (c, c), (b, a), (c, a)}

Which of the following is (are) reflexive, symmetric or transive?

Solution:

1. For R1 to be reflexive, for each x ∈ A, (x, x) ∈ R1 , i.e. (a, a), (b, b), (c, c) ∈
R1 . But in our case only (a, a) ∈ R1 . So R1 is not reflexive.

For R1 to be symmetric, we must have that for every (x, y) ∈ R1 , also


(y, x) ∈ R1 . We (a, b) ∈ R1 but (b, a) ∈
/ R1 . This is enough to conclude
that R1 is not symmetric. Or (b, c) ∈ R1 but (c, b) ∈
/ R1 . (you may not
show both situations unless specified).

For transitive, we must have for (x, y) ∈ R1 , (y, z) ∈ R1 then (x, z) ∈ R1 .


We have (a, b) ∈ R1 and (b, c) ∈ R1 , so for transitivity, we need (a, c) ∈ R1 .
But in this case, (a, c) ∈
/ R1 . So R1 is not transitive.

2. R2 is not reflexive since (a, a), (b, b), (c, c) ∈


/ R2 .
We have (a, b) ∈ R2 as well as (b, a) ∈ R2 and (a, c) ∈ R2 as well as
(c, a) ∈ R2 , hence R2 is symmetric.
(b, a) ∈ R2 and (a, c) ∈ R2 , but (b, c) ∈
/ R2 , hence R2 is not transitive.

3. R3 is not reflexive since (c, c) ∈


/ R3 , even though (a, a) ∈ R3 and (b, b) ∈
R3 . R3 is not symmetric since (a, c) ∈ R3 but (c, a) ∈
/ R3 , even though

48
(a, b), (b, a) ∈ R3 .
Since (a, b) ∈ R3 , (b, c) ∈ R3 and (a, c) ∈ R3 , then R3 is transitive.

4. Show that R4 is both reflexive, symmetric, and transitive.

Hence, only R4 is equivalence relation.

Example 2.3.10. Let R be a relation from R to R defined by xRy iff x − y is


divisible (multiple) of 10,i.e.

R = {(x, y) : x − y = 10k, for k ∈ Z}.

Show that R is an equivalence relation.


Solution:

(a) Reflexive:
We must show that xRx. Now x − x = 0 = 10 · k, hence divisible by 10.
So R is reflexive.

(b) Symmetric:
We must show that if xRy, then yRx. Suppose xRy, then

x − y = 10k, for k ∈ Z

−(y − x) = 10k

y − x = 10(−k)

But −k ∈ Z if k ∈ Z, hence y − x is also divisible by 10.


So R is symmetric.

(c) Transitive:
We must show that if xRy, and yRz, then xRz. If xRy, and yRz, then
x − y = 10k and y − z = 10h for some k, h ∈ Z.

49
Now,

x − z = (x − y) + (y − z)

= 10k + 10h

= 10(k + h)

Since k ∈ Z and h ∈ Z, then k + h ∈ Z. So x − z is divisible by 10.

2.3.2 Equivalence Classes and Partitions

Definition 2.3.11. Let R be an equivalence relation on the nonempty set A.


For a ∈ A, the equivalence class of a determined by the relation R is
the set
[a] = {b ∈ A : aRb}.

Example 2.3.12. Suppose A = {0, 1, 2, 3, 4, 5, 6, 7, 8}. Let R be a relation on A


defined by aRb iff a − b = 2k for k ∈ Z. It is easy to show that R is equivalence
relation. Now for any a ∈ A, [a] = {b ∈ A : aRb} is an equivalence class under
R. For instance

[0] = {b ∈ A : 0Rb} = {b ∈ A : 0−b = 2k, k ∈ Z} = {b ∈ A : b = 2(−k), k ∈ Z} = {0, 2, 4, 6, 8};

[1] = {b ∈ A : 1Rb} = {b ∈ A : 1−b = 2k, k ∈ Z} = {b ∈ A : b = 1−2k, k ∈ Z} = {1, 3, 5, 7};

[2] = {b ∈ A : 2Rb} = {b ∈ A : 2−b = 2k, k ∈ Z} = {b ∈ A : b = 2−2k, k ∈ Z} = {0, 2, 4, 6, 8};

[3] = {b ∈ A : 3Rb} = {b ∈ A : 3−b = 2k, k ∈ Z} = {b ∈ A : b = 3−2k, k ∈ Z} = {1, 3, 5, 7};

[4] = {b ∈ A : 4Rb} = {b ∈ A : 4−b = 2k, k ∈ Z} = {b ∈ A : b = 4−2k, k ∈ Z} = {0, 2, 4, 6, 8};

[5] = {b ∈ A : 5Rb} = {b ∈ A : 5−b = 2k, k ∈ Z} = {b ∈ A : b = 5−2k, k ∈ Z} = {1, 3, 5, 7};

[6] = {b ∈ A : 6Rb} = {b ∈ A : 6−b = 2k, k ∈ Z} = {b ∈ A : b = 6−2k, k ∈ Z} = {0, 2, 4, 6, 8};

[7] = {b ∈ A : 7Rb} = {b ∈ A : 7−b = 2k, k ∈ Z} = {b ∈ A : b = 7−2k, k ∈ Z} = {1, 3, 5, 7};

[8] = {b ∈ A : 8Rb} = {b ∈ A : 8−b = 2k, k ∈ Z} = {b ∈ A : b = 8−2k, k ∈ Z} = {0, 2, 4, 6, 8};

50
2.3.3 Congruence Modulo

Let R be an equivalence relation, if xRy we say “x is congruent to y modulo


n” whenever n ∈ N is a divisor of x − y.We write x ≡ y( mod n).

Example 2.3.13. Solve for x if

x ≡ 5( mod 3).

Solutions:

x ≡ 5( mod 3) ⇒ x − 5 = 3k for k ∈ Z.
Then x = 3k + 5. So for k ∈ {. . . , −3, −2, −1, 0, 1, 2, 3, . . .}, we get

x = {. . . , −4, −1, 2, 5, 8, 11, 14, . . .}.

This problem is a consequence of finding [5] when we define relation R to by


xRy iff x − y = 3k for k ∈ Z, hence

[5] = {b ∈ Z : bR5} = {b ∈ A : b − 5 = 3k, k ∈ Z}

= {b ∈ Z : b = 3k + 5, k ∈ Z}

= {. . . , −4, −1, 2, 5, 8, 11, 14, . . .}.

Example 2.3.14. Given R on Z. Define xRy by x + y is even integer.

(a) Show that R is an equivalence relation on Z.

(b) Find [0] and [3].

Solutions:

(a)

(i) Since x + x = 2x and since x ∈ Z then 2x is even, hence xRx hence R is


reflexive.

(ii) If xRy, then x + y is even hence y + x is still even since addition in Z is


commutative, so yRx, hence R is symmetric.

51
(iii) Let xRy and yRz, so x + y = 2k and y + z = 2r for k, r ∈ Z. Adding the
two equations we get

(x + y) + (y + z) = 2k + 2r

x + z = 2k + 2r − 2y

= 2(k + r − y)

hence even since k + r − y ∈ Z. So xRz is also satisfied hence R is


transitive. In conclusion R is an equivalence relation.

(b)

[0] = {y ∈ Z : 0Ry} = {y ∈ Z : 0 + y = 2k k ∈ Z}

= {y ∈ Z : y = 2k k ∈ Z}

= {. . . , −4, −2, 0, 2, 4, . . .}

[3] = {y ∈ Z : 3Ry} = {y ∈ Z : 3 + y = 2k, k ∈ Z}

= {y ∈ Z : y = 2k − 3, k ∈ Z}

= {. . . , −7, −5, −3, −1, 1, . . .}

Example 2.3.15. Define a relation R on R × R by:

(x, y)R(u, v) iff x2 + y 2 = u2 + v 2 .

(a) Prove that R is an equivalence relation on R × R.

(b) Give a description of the members of each of the following equivalence


classes:
[(0, 0)]; [(1, 1)]; [(3, 4)].

(c) Describe [(a, b)] for any fixed values a, b both in set-theoretic terms as well
as geometrically.

52
Solution:

(a) reflexive:
(x, y)R(x, y) By definition x2 + y 2 = x2 + y 2 , so R is clearly reflexive.
symmetric:

(x, y)R(u, v) ⇒ x2 + y 2 = u2 + v 2

x2 − u2 = −y 2 + v 2

−u2 − v 2 = −x2 − y 2

−(u2 + v 2 ) = −(x2 + y 2 )

u2 + v 2 = x2 + y 2

⇒ (u, v)R(x, y)

Therefore R is symmetric. transitive:


Let (x, y)R(u, v) and (u, v)R(s, t). We need to show that (x, y)R(s, t)
Now, (x, y)R(u, v) iff x2 +y 2 = u2 +v 2 and (u, v)R(s, t) iff u2 +v 2 = s2 +t2 .
By the property of “=” we deduce that x2 + y 2 = s2 + t2 ⇒ (x, y)R(s, t)
and thus R is transitive. Hence R is an equivalence relation.

(b)

[(0, 0)] = {(x, y) ∈ R × R : (x, y)R(0, 0)}

= {(x, y) ∈ R × R : x2 + y 2 = 02 + 02 }

= {(x, y) ∈ R × R : x2 + y 2 = 0}

which is a circle with centre origin and radius 0.

[(1, 3)] = {(x, y) ∈ R × R : (x, y)R(1, 3)}

= {(x, y) ∈ R × R : x2 + y 2 = 12 + 32 }

= {(x, y) ∈ R × R : x2 + y 2 = 10}

53

which is a circle with centre origin and radius 10.

[(8, 6)] = {(x, y) ∈ R × R : (x, y)R(8, 6)}

= {(x, y) ∈ R × R : x2 + y 2 = 82 + 62 }

= {(x, y) ∈ R × R : x2 + y 2 = 100}

which is a circle with centre origin and radius 10.

(c)

[(a, b)] = {(x, y) ∈ R × R : (x, y)R(a, b)}

= {(x, y) ∈ R × R : x2 + y 2 = a2 + b2 }

which is a family of concentric circles centred at the origin.

which is a family of concentric circles centred at the origin.

Tutorials 2.3.16. 1. Determine whether the following relations are (i) re-
flexive, (ii) symmetric, (iii) transitive.

(a) Let A = {a, b, c} define a relation or R, where

R = {(b, b), (c, a), (a, b), (a, c), (b, c), (b, a), (c, b), (c, c)}.

(b) Relation R defined on Z where (x, y) ∈ R iff x − y = 4.

(c) Relation R defined on N where (m, n) ∈ R iff m divides n.

(d) Relation R defined on R where xRy iff x < y.

(e) Let R be a relation on cartesian product N × N where (x, y)R(u, v)


iff xv − yu = 1

2. From the previous question above, which of (a) to (d) are equivalence re-
lation? Give reasons based on on what you shown above.

54
3. Show that the following are equivalence relation.

(a) Let A = {a, b, c, d, e} define a relation or R, where


R = {(b, a), (b, b), (e, a), (a, a), (e, d), (a, d), (a, e), (b, d)(c, c), (d, a), (e, e), (c, b),
(d, d), (a, b), (d, e)},

(b) Relation R defined on Z − {0} where (x, y) ∈ R iff xy > 0.

(c) Relation R defined on Z where (x, y) ∈ R iff x − y = 7k for k ∈ Z.

(d) Let R be a relation on cartesian product N × N where (x, y)R(u, v)


iff xv = yu

4. Find the equivalence classes of the following using question 3 above

(a) [b] and [e] in (a).

(b) [3] and [7] in (b).

(c) [0] and [5] in (c).

(d) [(3, 5)] and [(1, 1)] in (d).

2.4 Algebraic structures


A binary operation ◦ on a set A is a map ◦ : A×A → A. Using such an operation
we shall often write a ◦ b (or a ◦ b) instead of ◦(a, b): Note the difference:

• when R : A → B is a relation, aRb is an assertion that is true if and only


if (a; b) is in R;

• when ◦ : A × A → A is an operation and a and b are in A, a ◦ b is an


element in A.

Remarks 2.4.1. • N is the set of natural numbers, the most familiar clas-
sical operations on it are addition + and multiplication ×, more often
written as ·, and one even writes ab instead of a · b:

• Z is the set of integers; apart from the addition and multiplication, denoted
by the same symbols as for natural numbers, one of the most important

55
binary operations on it is subtraction −: Note that here we use ”minus”
as a binary operation, although it can also be used as a unary operation,
that is, a map A → A.

• The set Q of rational numbers, the set R of real numbers, and the set C of
complex numbers all have addition, multiplication, and subtraction defined
on them, and denoted by the same symbols as for integers.

2.4.1 Associativity, Commutativity and idempotency

Many binary operations ◦ : A × A → A satisfy one or more of the following


identities:

• x ◦ (y ◦ z) = (x ◦ y) ◦ z (associativity),

• x ◦ y = y ◦ x (commutativity),

• x ◦ x = x (idempotency); for all for all x, y, z ∈ A, and called associative,


commutative, idempotent respectively.

Tutorials 2.4.2. (1) Which of the followings define a binary operations on the
set of integers? Of those that do, which are associative? Which are commuta-
tive? Which are idempotent?
(a) m ◦ n = mn + 1,
(b) m ◦ n = (m + n) ÷ 2,
(c) m ◦ n = m,
(d) m ◦ n = mn2 ,
(e) m ◦ n = m2 + n2 ,
(f ) m ◦ n = 3,
(g) m ◦ n = mn .
(2) Let m be a binary operation on the set of real numbers defined by x ◦ y =
ax + by + c.
(a) Find the set of all triples (a, b, c) of real numbers, for which ◦ is associative.
(b) Find the set of all triples (a, b, c) of real numbers, for which ◦ is commuta-
tive.

56
(c) Find the set of all triples (a, b, c) of real numbers, for which ◦ is idempotent.
(3) Find all binary operations on the set {0, 1}.

2.4.2 Algebraic structures

Remarks 2.4.3. Intuitively, a mathematical structure is a pair (X, s), where


X is a set, called the underlying set of the structure (X, s), and s; the struc-
ture itself, is anything defined on X. Sometimes we shall write just X instead
of (X, s), or, say, if s itself is a pair (s1 , s2 ), then consider (X, s) as the triple
(X, s1 , s2 ), etc.

0 0
An isomorphism f : (X, s) → (X , s ) of structures of the same type is a bi-
0
jection f : X → X such that both f and its inverse bijection preserve the struc-
0 0
ture. When such an isomorphism exist, we also say that (X, s) and (X , s ) are
isomorphic to each other. We shall not give any general definition, but only
consider a few examples:

2.4.3 Examples of algebraic structures

1. A magma is a pair (X, ◦), where X is a set and ◦ is a binary operation


0 0
on it. An isomorphism f : (X, ◦) → (X , ◦ ) of magmas is a bijection f :
0 0
XrightarrowX with f (x ◦ y) = f (x) ◦ f (y) ∗
f orallxandy in X: Note that ∗ implies that the bijection g; inverse of f ; has
0 0 0
the same property. Indeed, for every x and y in X , we have
0 0 0 0 0 0 0 0 0 0 0 0
g(x ◦ y ) = g(f g(x ) ◦ f g(y )) = gf (g(x ) ◦ g(y )) = g(x ) ◦ g(y ).

Sometimes one uses either the multiplicative notation, writing x · y (or simply
xy) instead of x ◦ y, or the additive notation, writing x + y, and then calls
magmas multiplicative or additive, respectively. If so, ∗ becomes

f (xy) = f (x)f (y) or f (x + y) = f (x) + f (y),

accordingly. In old literature magmas were called groupoids.

57
2. A magma (X, ◦) is said to be associative, commutative, idempotent, if so is
the operation ◦, respectively. An associative magma is also called a semigroup,
and, accordingly, an associative commutative magma is called a commutative
semigroup, and term ”idempotent” is used in the same way. One also says that
an element x of a magma (X, ◦) is idempotent if x ◦ x = x, according to this
terminology, a magma (X, ◦) is idempotent if and only if so is every element of X.

3. A unitary magma is a triple (X, e, m), in which (X, m) is a magma and


e an element in X with m(e, x) = x = m(x, e) for every x in X: When either
the multiplicative, or the additive notation is used, one writes 1 or 0 instead of
e and

1x = x = x1 or 0 + x = x = x + 0,

respectively. The element e above sometimes called

• the neutral element’ or

• the identity element or the unit, in the case of multiplicative notation, or

• the zero element or just zero, in the case of additive notation.

Note that such e is uniquely determined by m: Moreover, for any magma (X; m)
0
and elements e and e in X, we have:
0 0
if m(e, x) = x and m(x, e ) = x for every x in X; then e = e , proving that the
identity is uniquely determined by m.
0 0
Indeed, we can write e = m(e, e ) = e . This allows us to define an isomor-
0 0 0
phism f : (X, e, m) → (X , e , m ) of unitary magmas as simply an isomorphism
0 0 0
f : (X, m) → (X , m ) of magmas. Since the equality f (e) = e , which we
obviously would like to have, can be proved for any such f .

4. A unitary magma (X, e, m) is said to be associative, commutative, idem-


potent, if so is the magma (X, m), respectively. An associative unitary magma

58
is called a monoid. Just like a semigroup, a monoid can be commutative or
idempotent.

5. A group is a quadruple (X, e, m, i) for which (X, e, m) is a monoid and i


is a unary operation on X with m(x, i(x)) = e = m(i(x), x). In the multiplica-
tive notation one writes x−1 instead of i(x) and the equation above becomes
xx−1 = 1 = x−1 x, while in the additive notation i(x) is written as x and the
equation becomes x + (−x) = 0 = (−x) + x. Here again, the new operation
is uniquely determined by the previously introduced ones, in the sense that if
0 0
(X, e, m, i) and (X, e, m, i ) are groups, then i = i . Moreover, for any monoid
(X, e, m) and elements x, y, z ∈ X; we have:

if m(x, y) = e = m(z, x) then y = z.

Indeed, the equalities m(x, y) = e = m(z, x) imply

y = m(e, y) = m(m(z, x), y)) and z = m(z, e) = m(z, m(x, y)),

while m(m(z, x), y)) = m(z, m(x, y)) by the associativity of m : The same argu-
ment looks even simpler with either the multiplicative, or the additive notation.
Using, say, the multiplicative notation we write

y = 1y = (zx)y = z(xy) = z1 = z.
0 0 0 0
And again, this allows us to define an isomorphism f : (X, e, m, i) → (X , e , m , i )
0 0
of groups as simply an isomorphism f : (X; m) → (X ; m ) of magmas, since
0 0
not only the equality f (e) = e , but also f i = i f ; can be proved for any such
f . A group (X, e, m, i) is said to e commutative ot abelian if m is commutative.

6. A ring (R, +, ·, 0, 1) is a set R with two binary operations, additions and


multiplication, and a nullary operation, ”select 1”, such that:
(i) (R, +, 0) is an abelian group.
(ii) (R, ·, 1) is a monoid.
(iii) Multiplication is distributive (on both sides) over addition. The last re-
quirement means that all triples of elements a, b, c ∈ R satisfy the identities

59
a(b + c) = ab + ac, (a + b)c = ac + bc.

A commutative ring is one in which the multiplication is commutative. An


isomorphism f : (R, +, ·, 0, 1) → (R0, +, ·, 0, 1) of rings is simply a bijection
0
f :R→R.

7. A vector space is an additive abelian group whose elements can be suit-


ably multiplied by the elements from some field F of ”scalars”. The formal
definition is as follows: A vector space X is an additive abelian group together
with a map F × X → X, written (a, x) 7→ ax, and subject to the following
axioms, for all elements a, b ∈ F and a, b ∈ X : a(a + b) = aa + ab,
(a + b)a = aa + ba,
(ab)a = a(ba),
1a = a.

Tutorials 2.4.4. Consider all possible associative binary operations on the set
{0, 1} and find those that make the set {0, 1} :
(a) a unitary magma,
(b) a semigroup,
(c) a monoid,
(d) a group.

2.4.4 Boolean algebras

A Boolean algebra is a mathematical structure (B, 0, ∨, 1, ∧, ¬), in which B is


a set, 0 and 1 are elements of B, ¬ is unary operation on B, and ∨ and ∧ are
binary operations on B; satisfying the following equalities
a ∨ (b ∨ c) = (a ∨ b) ∨ c
a ∨ b = b ∨ a,
a ∨ a = a,
a ∧ (b ∧ c) = (a ∧ b) ∧ c,
a ∧ b = b ∧ a,
a ∧ a = a,

60
a ∧ (a ∨ b) = a,
a ∨ (a ∧ b) = a,
a ∧ (b ∨ c) = (a ∧ b) ∨ (a ∧ c)
a ∨ (b ∧ c) = (a ∨ b) ∧ (a ∨ c)
a ∧ (¬a) = 0,
a ∨ (¬a) = 1,
0 ∨ a = a = a ∨ 0,
and 1 ∧ a = a = a ∧ 1,
for all a, b, c ∈ B. Using the same symbols 0∨, 1, ∧, ¬ for all Boolean algebras, we
0
define an isomorphism f : (B, 0∨, 1, ∧, ¬) → (B , 0∨, 1, ∧, ¬) of Boolean algebras
0
(obviously) as a bijection f : B → B with f (0) = 0, f (a ∨ b) = f (a) ∨ f (b),
f (1) = 1, f (a ∧ b) = f (a) ∧ f (b), f (¬a) = ¬f (a), for all a, b ∈ B.

Tutorials 2.4.5. 1. Let (B, 0∨, 1, ∧, ¬) be a Boolean algebra and x; y ∈ B.


Prove the following equalities: (a) ¬(x ∨ y) = ¬x ∧ ¬y.
(b) ¬(x ∧ y) = ¬x ∨ ¬y.
(c) x ∨ y = ¬(¬x ∧ ¬y).
(d) x ∧ y = ¬(¬x ∨ ¬y).
(e) ¬0 = 1.
(f ) ¬1 = 0.
(g) x ∧ (¬x ∨ y) = x ∧ y.
(h) x ∨ (¬x ∧ y) = x ∨ y.
0
2. Let f : (B, 0∨, 1, ∧, ¬) → (B 0∨, 1, ∧, ¬) be an isomorphism. Prove that: (a)
f (0) = 0 and f (1) = 1
(b) f (x ∨ y) = f (x) ∨ f (y).

2.5 Functions
Definition 2.5.1. Let A and B be sets. A function f from A to B, denoted
by f : A → B, is a relation from A to B such that

(a) Domain of f is A

61
(b) If (a, b) ∈ f and (a, c) ∈ f , then b = c.

The domain of f from A to B is the set

Dom(f ) = {x ∈ A : (∃y ∈ B)((x, y) ∈ f )}

The codomain of the function from A to B is the set B.


The range of a function f from A to B is the set

Rng(f ) = {y ∈ B : (∃x ∈ A)((x, y) ∈ f )}

2.5.1 One-to-One and Onto functions

Onto(Surjective) Functions

Definition 2.5.2. Let A and B be sets. A function f : A → B is onto if and


only if
∀y ∈ B, ∃x ∈ A such that f (x) = y.

Example 2.5.3. (a) Show that function f : R → R defined by f (x) = 5x − 2


is onto.

Solution:

Let y ∈ R be the codomain R. We must find an x in the domain R such


that
f (x) = 5x − 2 = y. By solving for x we find,

5x − 2 = y

5x = y + 2
y+2
x= ∈ R for any y ∈ R.
5

(b) Show that function f : R → R defined by f (x) = x2 + 4 is not onto.

62
Solution:

Since the codomain is any y ∈ R, we can prove by using counter-example


that x cannot exist for all R. Let y = 2, then

x2 + 4 = 2 ⇒ x2 = −2.

It is clear from this that there is no solution of x which satisfies such


expression.
Actually x2 + 4 ≥ 4 i.e. the minimum value is 4.

One-to-One(Injective) Functions

Definition 2.5.4. Let A and B be sets. A function f : A → B is one-to-one


(injective) if and only if for
f (x) = f (y) ⇒ x = y, for x, y ∈ A or if x 6= y ⇒ f (x) 6= f )y).

Example 2.5.5. (a) Show that function f : R → R defined by f (x) = 5x − 2


is one-to-one.

Solution:

Assume that f (x) = f (y). We must check if x = y. Then

f (x) = f (y) ⇒ 5x − 2 = 5y − 2

5x = 5y

x = y.

(b) Show that function f : R → R defined by f (x) = x2 + 4 is not one-to-one.

Solution:

Let’s attempt to prove that f (x) is one-to-one. Then

f (x) = f (y) ⇒ x2 + 4 = y 2 + 4

x2 = y 2 .

63
But the last expression doesn’t mean that x = y, for example, if x = 3
and y = −3. It is clear that x2 = y 2 = 9. So f (x) = f (y) = 13 but
3 = x 6= y = −3, hence f is not one-to-one.

Tutorials 2.5.6. Check whether the following functions are onto or one-to-one.
Show your steps clearly.

(i) f : R → R defined by f (x) = −3x + 2

(ii) f : R → R defined by f (x) = −x2 − 5

(iii) f : R+ → R+ defined by f (x) = x2

(iv) f : R → [−1, 1] defined by f (x) = cos x

2.5.2 Inequalities and absolute value

Example 2.5.7. Solve for x: 3x2 + 10x ≤ 5x + 2.

3x2 + 10x ≤ 5x + 2

3x2 + 10x − 5x − 2 ≤ 0

3x2 + 5x − 2 ≤ 0

(3x − 1)(x + 2) ≤ 0

1
Critical values are: x = 3 and x = −2.
We can use the table or number line method to find our solution:

1 1 1
x < −2 x = −2 −2 < x < 3 x= 3 x> 3

3x − 1 − − − 0 +
x+2 − 0 + + +
(3x − 1)(x + 2) + 0 − 0 +

From this table it is clear that

1
−2 ≤ x ≤ .
3

64
3x−1
Example 2.5.8. Solve for x in x+2 ≥ 2.

3x − 1
≥2
x+2
3x − 1
−2≥0
x+2
3x − 1 − 2(x + 2)
≥0
x+2
3x − 1 − 2x − 4
≥0
x+2
x−5
≥ 0 (x 6= −2)
x+2

Critical values are: x = 5 and x = −2.


We can use the table or number line method to find our solution:

x < −2 x = −2 −2 < x < 5 x=5 x>5


x−5 − − − 0 +
x+2 − 0 + + +
3x−1
x+2 + ∞ − 0 +

From the table we have solution:

x < −2 or x ≥ 5, i.e.(−∞, −2) ∪ [5, ∞).

Example 2.5.9. Solve for x: |3x − 4| > x + 1.

⇒ 3x − 4 > x + 1 or − (3x − 4) > x + 1

⇒ 3x − x > 1 + 4 or − 3x + 4 − x > 1

⇒ 2x > 5 or − 4x > −3
5 3
⇒x> or x <
2 4

So from this we conclude


3 5
(−∞, ) ∪ ( , ∞).
4 2
|x|−3
Example 2.5.10. Solve for x in x−2 < 2.

65
Case 1:

If x > 2.

|x| − 3
< 2 ⇒ |x| − 3 < 2(x − 2)
x−2
⇒ |x| < 2x − 4 + 3

⇒ |x| < 2x − 1

⇒ −(2x − 1) < x < 2x − 1

⇒ −2x + 1 < x < 2x − 1

⇒ −2x + 1 < x and x < 2x − 1

⇒ −3x < −1 and − x < −1


1
⇒x< and x > 1
3
⇒ x > 1.

Case 2:

If x < 2.

|x| − 3
> 2 ⇒ |x| − 3 > 2(x − 2)
x−2
⇒ |x| > 2x − 4 + 3

⇒ |x| > 2x − 1

⇒ x > 2x − 1 OR − x > 2x − 1

⇒ −x > −1 OR − 3x > −1
1
⇒ x < 1 OR x <
3

So from this we conclude


1
(−∞, ) ∪ (1, ∞).
3
Tutorials 2.5.11. In each of the following, solve for x.

1. (x + 2)2 < 4

66
4x+1
2. x−1 ≥1

3. |5x − 1| ≤ 2x − 3

x+3
4. 2x−1 <1

|x|+5
5. x−3 <2

“God does arithmetic”. Carl Friedrich Gauss

67
Chapter 3

MATHEMATICAL
INDUCTION, BINOMIAL
THEOREM AND
COMBINATORICS

”God used beautiful mathematics in creating the world”. Paul Dirac

3.1 Mathematical Induction


Definition 3.1.1. Natural numbers are the numbers used in counting. Thus
we denote the set of natural numbers as:N = {1, 2, 3, 4, . . .}.

The Principle of Mathematical Induction is a method of proving results about


the natural numbers or positive integers. This is based on the fact that the
natural numbers can be arranged in a sequence, with a definite starting point
1, and in such case, each natural number has a unique successor.

68
The Principle of Mathematical Induction:

Let P (n) be a predicate on the set of natural numbers N satisfying:

(i) P (1) is a true statement;

(ii) If P (k) is true ⇒ P (k + 1) is also true for some k ∈ N,

then P (n) is true for all natural numbers.

To see how this works, let’s go through some examples.

Example 3.1.2. Use the Principle of Mathematical Induction to prove that:

n2 (n + 1)2
1 3 + 2 3 + 3 3 + . . . + n3 = ∀n ∈ N.
4
Solution:
n2 (n+1)2
Let P (n) be the statement 13 + 23 + 33 + . . . + n3 = 4 ∀n ∈ N.
3 12 (1+1)2 4
P (1) : 1 = 1 = 4 = 4,

hence P (1) is true.


k2 (k+1)2
Assume P (k) is true i.e. 13 + 23 + 33 + . . . + k 3 = 4 for some k ∈ N.
Then we need to deduce that P (k + 1) is also true, i.e.

(k + 1)2 [(k + 1) + 1]2


13 + 23 + 33 + . . . + k 3 + (k + 1)3 =
4
(k + 1)2 (k + 2)2
=
4
Now

LHS = 13 + 23 + 33 + . . . + k 3 + (k + 1)3
k 2 (k + 1)2
= + (k + 1)3 (by assumption),
4
k2
= (k + 1)2 [ + (k + 1)] (taking common factor of (k + 1)2 ),
4
2
k + 4k + 4
= (k + 1)2 [ ]
4
(k + 2)(k + 2)
= (k + 1)2 [ ]
4
(k + 1)2 (k + 2)2
=
4
= RHS.

69
Thus P (k + 1) is also true.
Hence P (n) is true ∀n ∈ N.

Example 3.1.3. Use the Principle Mathematical Induction to prove that 7n+2 +
82n+1 is divisible by 57 for every non-negative integer, i.e. n ≥ 0.
Solution:
Let P (n) be the statement: 7n+2 +82n+1 is divisible by 57 for every non-negative
integer.
P (0) : 70+2 + 82(0)+1 = 49 + 8 = 57 which is divisible by 57.
Hence P (0) is true.
Assume P (k) is true for some k ≥ 0.i.e. 7k+2 + 82k+1 = 57p for p ∈ Z and
k ≥ 0.
Also
7k+2 = 57p − 82k+1 .

Then we need to deduce that P (k + 1) is also true, i.e.

7(k+1)+2 + 82(k+1)+1 = 7k+3 + 82k+3 = 57r for r ∈ Z

LHS = 7k+3 + 82k+3 ,

= 7(k+2)+1 + 8(2k+1)+2

= 7k+2 · 7 + 82k+1 · 82

= (57p − 82n+1 ) · 7 + 64 · 82k+1 by substituting 7k+2 = 57p − 82n+1 from assumption.

= 399p − 7 · 82k+1 + 64 · 82k+1 ,

= 399p + 57 · 82k+1

= 57(7p + 82k+1 )

= 57r with r = 7p + 82k+1 ∈ Z.

Thus P (k + 1) is also true.


Hence P (n) is true ∀n ≥ 0.

70
Definition 3.1.4. Factorial: For a natural number n, n factorial, written
n!, is defined as the product of the first n natural number, i.e.

n! = n(n − 1)(n − 2)(n − 3) . . . (3)(2)(1).

We also define 0! = 1
e.g. 7! = (7)(6)(5)(4)(3)(2)(1) = 5040.

From the definition we can easily deduce that:

n! = n(n − 1)! = n(n − 1)(n − 2)!

etc

Example 3.1.5. Use mathematical induction to prove that 2n < n! for n ≥ 4.


Solution:
Let P (n) be the statement:2n < n! for n ≥ 4.
P (4) : 24 = 16 < 4! = 24.
Hence P (4) is true.
Assume P (k) is true for some k ≥ 4.i.e. 2k < k! for k ≥ 4.
Then we need to deduce that P (k + 1) is also true, i.e. 2k+1 < (k + 1)!

From assumption

2 · 2k < 2k!

2k+1 < 2k!

< (4 + 1)k! since 2 < 4 + 1

≤ (k + 1)k! since it is given that 4 ≤ k

= (k + 1)!.

Thus P (k + 1) is also true.


Hence P (n) is true ∀n ≥ 4.

71
Tutorials 3.1.6. Use the Principle of Mathematical Induction to prove that:

(a) 1 + 3 + 5 + · · · + (2n − 1) = n2 for all n ∈ N.

n(n+1)(2n+1)
(b) 12 + 22 + 32 + · · · + n2 = 6 for all n ∈ N.

n(n+1)(n+2)
(c) 1 · 2 + 2 · 3 + 3 · 4 + · · · + n(n + 1) = 3 for all n ∈ N.

1 2 3 n 1
(d) 2! + 3! + 4! + ··· + (n+1)! =1− (n+1)! for all n ∈ N.

1 1 1 1 n−2
(e) 2·3 + 3·4 + 4·5 + ··· + (n−1)n = 2n for all n ≥ 3, n ∈ N.
h  n i
(f) 1 + 5−1 + 5−2 + · · · + 5−n+1 = 45 1 − 15 ∀n ∈ N.

(g) 5n − 4n − 1 is divisible by 16 ∀n ∈ N.

(h) 4n+1 + 52n−1 is divisible by 21 ∀n ∈ N.

(i) n2 + n is even ∀n ∈ N.

(j) 2n > n2 for all n > 4.

(k) 3n < n! for all n > 6.

(l) 1 + 2n ≤ 3n for all ∀n ∈ N.

(m) n7 − n is divisible by 7 ∀n ∈ N (Hint: Use Binomial theorem to expand.)

3.2 Factorials and Binomial Expansions


Recall the factorial of the natural number n! = n(n−1)(n−2)(n−3) . . . (3)(2)(1).

Example 3.2.1. Simplify the following:

24! (24)(23)(22)(21)20!
(a) 20! = 20! = (24)(23)(22)(21) = 255024.

(n+1)! (n+1)n(n−1)!
(b) (n−1)! = (n−1)! = n(n + 1) = n2 + n

72
3.2.1 Binomial expansions and Binomial theorem

Definition 3.2.2. Binomial coefficient: Let n, r ∈ N, such that 0 ≤ r ≤ n.


Then  
n n!
= ;
r r!(n − r)!
where nr is called the combinatorial symbol which can also be written as n Cr .


n n
 
r is read as “n choose r”. We interpret r as a number of ways of choosing

r objects from the total of n objects.

9 9! 9! 9·8·7·6!

Example 3.2.3. 3 = 3!(9−3)! = 3!6! = 3·2·1·6! = 84.

Theorem 3.2.4. For n, r ∈ N and 0 ≤ r ≤ n. The following condition holds:

n n
 
(i) 0 = n = 1.

n n
 
(ii) 1 = n−1 = n.

n n
 
(iii) r = n−r .

n n! n!

Proof. (i) 0 = 0!(n−0)! = n! = 1. We used the fact that 0! = 1.
n n! n!

n = n!(n−n)! = n!0! =1

n
 n! n(n−1)!
(ii) 1 = 1!(n−1)! = (n−1)! = n.
n
 n! n(n−1)!
n−1 = (n−1)![n−(n−1)]! = (n−1)!1! = n.

(iii) will be done as an exercise.




Lemma 3.2.5. Pascal Rule: For n, r ∈ N and 0 ≤ r ≤ n;


     
n n n+1
+ =
r r−1 r

or      
n n n+1
+ = .
r r+1 r+1

73
n n n+1
  
Proof. We will show the proof of r + r−1 = r .

   
n n n! n!
+ = +
r r−1 r!(n − r)! (r − 1)![n − (r − 1)]!
n! n!
= +
r!(n − r)! (r − 1)!(n − r + 1)!
n! n!
= +
r(r − 1)!(n − r)! (r − 1)!(n − r + 1)(n − r)!
n! h1 1 i
= +
(r − 1)!(n − r)! r n−r+1
n! hn − r + 1 + ri
=
(r − 1)!(n − r)! r(n − r + 1)
n!(n + 1)
=
(r − 1)!r(n − r)!(n − r + 1)
(n + 1)!
=
r!(n − r + 1)!
 
n+1
=
r

Example 3.2.6. Evaluate (a) 15 15 24 23


   
7 + 8 and (b) 10 − 9 .

Solution: (a) 15 15 15+1


= 16 16! 16!
   
7 + 8 = 8 8 = 8!(16−8)! = 8!8! = 12 870.

(b) First note that 23 23 24


  
9 + 10 = 10 , hence
24 23 23 23! 23!
  
10 − 9 = 10 = 10!(23−10)! = 10!13 = 1 144 066.

Pascals rule is used to generate the coefficients in binomial expansions. Consider


the patterns that follows below where each row has a value of n starting from
row 1 with n = 0.

74
 
0
0
   
1 1
0 1
     
2 2 2
0 1 2
       
3 3 3 3
0 1 2 3
         
4 4 4 4 4
0 1 2 3 4
           
5 5 5 5 5 5
0 1 2 3 4 5
             
6 6 6 6 6 6 6
0 1 2 3 4 5 6
               
7 7 7 7 7 7 7 7
0 1 2 3 4 5 6 7
..
.

The coefficients give rise to a special pattern which is called the Pascal’s Tri-
angle (after French mathematician Blaise Pascal).

1 1

1 2 1

1 3 3 1

1 4 6 4 1

1 5 10 10 5 1

1 6 15 20 15 6 1

1 7 21 35 35 21 7 1
..
.

75
We can use this to get coefficient of a binomial expression.

(x + y)0 = 1

(x + y)1 = x + y

(x + y)2 = x2 + 2xy + y 2

(x + y)3 = x3 + 3x2 y + 3xy 2 + y 3


..
.

(x + y)7 = x7 + 7x6 y + 21x5 y 2 + 35x4 y 3 + 35x3 y 4 + 21x2 y 5 + 7xy 6 + y 7 ,

and so forth.

Theorem 3.2.7. Binomial Theorem: Let x and y be variables, and let n, r ∈


N such that 0 ≤ r ≤ n. Then
n  
X n n−r r
(x + y)n = x y
r=0
r
       
n n n n−1 n n−1 n n
= x + x y + ... + xy + y .
0 1 n−1 n

Proof. We prove our result by using the principle of mathematical induction.


Pn
We let P (n) : (x + y)n = r=0 nr xn−r y r for n, r ∈ N and x, y ∈ R.

P1
P (1) : (x + y)1 = r=0 1r xn−r y r = 10 x + 11 y = x + y, which is true. Suppose
  

the result is true for some n = k, i.e. we assume


k  
X k
P (k) : (x + y)k = xk−r y r is true.
r=0
r

We must now deduce that P (k + 1) is also true, i.e. we must show that
k+1
X 
k+1 k + 1 (k+1)−r r
P (k + 1) : (x + y) = x y is also true.
r=0
r

76
(x + y)k+1 = (x + y)(x + y)k
k  
X k k−r r
= (x + y) x y by inductive hypothesis
r=0
r
k   k  
X k k−r r X k k−r r
=x x y +y x y
r=0
r r=0
r
k   k  
X k k−r+1 r X k k−r r+1
= x y + x y
r=0
r r=0
r
k   k+1  
X k k−r+1 r X k
= x y + xk−r+1 y r
r=0
r r=1
r − 1
k
X k   k  
k+1 k−r+1 r
X k
=x + x y + xk−r+1 y r + y k+1
r=1
r r=1
r − 1
k h   i
X k k
= xk+1 + + xk−r+1 y r + y k+1
r=1
r r − 1
k h 
X k + 1 i k−r+1 r
= xk+1 + x y + y k+1 , (Pascal Rule)
r=1
r
k+1
X k + 1

= xk−r+1 y r
r=0
r

So P (k + 1) is also true, hence our result is true. 

Example 3.2.8. Use the Binomial theorem to expand


(a) (a + b)6 and (b) (2x − 3y 2 )5 .
Solution:

6  
X 6
(a) (a + b)6 = a6−r br
r=0
r
             
6 6 6 5 6 4 2 6 3 3 6 2 4 6 5 6 6
= a + a b+ a b + a b + a b + ab + b
0 1 2 3 4 5 6
= a6 + 6a5 b + 15a4 b2 + 20a3 b3 + 15a2 b4 + 6ab5 + b6 .

77
(b) (2x − 3y 2 )5
5  
X 5
= (2x)5−r (−3y 2 )r
r=0
r
       
5 5 5 4 2 5 3 2 2 5
= (2x) + (2x) (−3y ) + (2x) (−3y ) + (2x)2 (−3y 2 )3
0 1 2 3
   
5 2 4 5
+ (2x)(−3y ) + (−3y 2 )5
4 5
= 32x5 + 5(16)(−3)x4 y 2 + 10(8)(9)x3 y 4 + 10(4)(−27)x2 y 6 + 5(2)(81)xy 8 + (−243)y 10

= 32x5 − 240x4 y 2 + 720x3 y 4 − 1080x2 y 6 + 810xy 8 − 243y 10 .

n

For a binomial expression (x+y)n , the r-th term is given by r−1 xn−r+1 y r−1 .

3y 10
Example 3.2.9. Find the 8th term of (x2 − x ) without expanding the ex-
pression.
Solution:
8th term of (x2 − 3y
x )
10
is
   
10 2 10−8+1 3y 8−1 10 3y
(x ) (− ) = (x2 )3 (− )7
8−1 x 7 x
10! y7
= (x6 )(−3)7 ( 7 )
7!(10 − 7)! x
10 · 9 · 8 · 7! 6−7
= x (−2187)y 7
7!3!
y7
= −262 440 .
x
Example 3.2.10. Without expanding the expression, find the coefficient of x5
from the binomial expression (x2 − x2 )10 .
Solution:
P10 10
Using the Binomial theorem, (x2 − x2 )10 = (x2 )10−r (− x2 )r .

r=0 r

We then simplify the general term:


   
10 2 10 20−2r
(x2 )10−r (− )r = x (−2)r x−r
r x r
 
10
= (−2)r x20−3r
r

78
Now to find the coefficient of x5 ,

x5 = x20−3r

5 = 20 − 3r

3r = 15

∴r=5

Therefore substituting r = 5, the coefficient of x5 is:

 
10 10!
(−2)5 = (−32) = −8 064.
5 5!5!
Example 3.2.11. Find the constant term of the binomial expression (2x+ x12 )9 .
P9
Using the Binomial theorem, (2x + x12 )9 = r=0 9r (2x)9−r ( x12 )r .


   
9 1 9 9−r 9−r −2r
(2x)9−r ( 2 )r = 2 x x
r x r
 
9 9−r 9−3r
= 2 x
r
Now the constant term is term independent of x hence the coefficient of x0 ,

x0 = x9−3r

0 = 9 − 3r

3r = 9

∴r=3
Therefore substituting r = 3, the constant term is:

 
9 9−3 9! 6
2 = 2 = 5 376.
3 6!3!
Another way of obtaining terms is by expressing each term as follows:

79
Example 3.2.12. Expand (a + b)8 .

T1 = xn = a8

T2 = nxn−1 y = 8a7 b
n(n − 1) n−2 2 8·7 6 2
T3 = x y = a b = 28a6 b2
2! 2·1
n(n − 1)(n − 2) n−3 3 8·7·6 5 3
T4 = x y = a b = 56a5 b3
3! 3·2·1
n(n − 1)(n − 2)(n − 3) n−4 4 8·7·6·5 4 4
T5 = x y = a b = 70a4 b4
4! 4·3·2·1
n(n − 1)(n − 2)(n − 3)(n − 4) n−5 5 8·7·6·5·4 3 5
T6 = x y = a b = 56a3 b5
5! 5·4·3·2·1
n(n − 1)(n − 2)(n − 3)(n − 4)(n − 5) n−6 6 8·7·6·5·4·3 2 6
T7 = x y = a b = 28a2 b6
6! 6·5·4·3·2·1
n(n − 1)(n − 2)(n − 3)(n − 4)(n − 5)(n − 6) n−7 7 8·7·6·5·4·3·2 7
T8 = x y = ab = 8ab7
7! 7·6·5·4·3·2·1
n(n − 1)(n − 2)(n − 3)(n − 4)(n − 5)(n − 6)(n − 7) n−8 8 8·7·6·5·4·3·2·1 0 8
T9 = x y = a b = b8
8! 8·7·6·5·4·3·2·1
Therefore, putting everything together we obtain:

(a + b)8 = a8 + 8a7 b + 28a6 b2 + 56a5 b3 + 70a4 b4 + 56a3 b5 + 28a2 b6 + 8ab7 + b8 .

3.2.2 Expansion of (a + b)n when n negative integer

If n is negative, the expansion of (a + b)n is infinite.

1
Example 3.2.13. Expand the first three terms of 3 3
(2x− x )
Solution:
We rewrite the expression as 1
3 3
(2x− x )
= (2x − x3 )−3
Comparing this with the binomial expansion (a + b)n , we substitute a = 2x,
b = − x3 and n = −3. We then obtain:
T1 = an = (2x)−3 = 1
8x3 ..

T2 = nan−1 b = (−3)(2x)−3−1 (− x3 ) = 9
16x4 x =9
16x5 .
2
n(n−1) n−2 2 (−3)(−3−1)
T3 = 2! a b = 2×1 (2x)−3−2 (− x3 )2 = (−3)(−4)(3)
2×25 x5 x2 = 27
16x7 .

So
1 3 −3 1 9 27
3 3 = (2x − x ) = 3+ 5
+
16x7
+ ...
(2x − x ) 8x 16x

80
3.2.3 Expansion of (a + b)n when n is a fraction

If n is a fraction, the expansion of (a + b)n is infinite.

8x3 − xy .
p
Example 3.2.14. Expand the first four terms of 3

Solution:
y 1
= (8x3 − xy ) 3 .
p
We rewrite the expression as 3
8x3 − x

Comparing this with the binomial expansion (a + b)n , we substitute a = 8x3 ,


b = − xy and n = 13 . We then obtain:
1
T1 = an = (8x3 ) 3 = 2x.
1
T2 = nan−1 b = 13 (8x3 ) 3 −1 (− xy ) = − y
2
y
= − (3)(4x y
2 )x = − 12x3 .
(3)(23 x3 ) 3 x
1 1
n(n−1) n−2 2 3 ( 3 −1) 3 31 −2 y2 2
T3 = 2! a b = 2·1 (8x ) (− xy )2 = − 19 5
y
= − 288x 7.
(23 x3 ) 3 x2
1 1 1 10
n(n−1)(n−2) n−3 3 3 ( 3 −1)( 3 −2)
1 y3
T4 = 3! x y = 3·2·1 (8x3 ) 3 −3 (− xy )3 = − 27
6 8 =
(23 x3 ) 3 x3
3
5y
− 20 736x 11 .

So

y2 5y 3
r
y y 1 y
3
8x3 − = (8x3 − ) 3 = 2x − 3
− 7
− − ...
x x 12x 288x 20 736x11

Tutorials 3.2.15. Show your workings in each of the following questions:

1. Give the following answers in binomial form.

77 77
 
(a) 31 + 32

92
− 91
 
(b) 22
21

(c) k+7 k+7


 
10 + 9

2. Use the binomial theorem to expand:

(a) (x + 3y)5

(b) (2x − y)6


 7
(c) x1 + 2x

3. Find the coefficient of x7 of the following expressions;

(a) (x − 2)10

81
 6
2
(b) x2 − x3
 7
1
(c) x4 − 2x

4. From the questions in 3. above, find the forth term of each in (a) to (c).
 5
5. Find the constant term of x22 − x3 .

6. Expand the first four terms of:

1
(a) (x−2)3
  13
2
(b) x2 − x3
 −2
y
(c) x − 2x2
2
(c)   21
2
x2 − yx

3.3 Combination and Permutations

3.3.1 Permutations

Suppose five people are standing in an ATM queue. the question is how many
possible orders are there of their standing position. To answer this, we must
first observe that there are five people, and the first person will be chosen in
five different ways. After choosing the first person we remain with four more
people and the second person will be chosen in four different ways. Continuing
in the same way we choose the third, fourth and fifth person in three, two and
one way(s) respectively. This gives us a total of

5 × 4 × 3 × 2 × 1 = 120 ways.

Suppose we were to select a team of 5 best athletes from 10 depending to their


finishing positions in a trial race. So the first runner will be selected in 10 ways,
the second in 9 ways, third in 8 ways, fourth in 7 ways and the fifth in 6 ways.
This gives us

10 × 9 × 8 × 7 × 6 = 30 240 ways of selecting the team.

82
In these examples we took into consideration of the order of arrangements. Such
arrangement were order is important is called permutation.

Theorem 3.3.1. The permutations, n Pr , of r objects from n is given by

n
Pr = n × (n − 1) × (n − 2) × · · · × (n − r + 1)

which can be written more compactly as

n n!
Pr = .
(n − r)!

In this way, we immediately see that the number of permutations of n objects


from n objects is
n n!
Pn = = n!.
0!
Example 3.3.2. How many two-digit numbers can be formed from the numbers
2, 3, 5, 7, 8 if

(a) there is no repetition?

(b) there is repetition?

Solution:

(a) With no repetition; There are five digits to choose from to get the two-
digit number. So the first digit can be chosen in 5 ways and second can
be chosen in 4 ways since we cannot repeat the digit already used. So we
have
5 × 4 = 20 ways.

(b) With repetition; the first digit can be chosen in 5 ways and second can be
again be chosen in 5 ways since we are allowed to repeat the digit already
used. So we have
5 × 5 = 25 ways.

Example 3.3.3. How many three-digit numbers can be formed from the num-
bers 0, 3, 4, 5, 6, 9 if

83
(a) there is no repetition?

(b) there is repetition?

Solution:

(a) With no repetition; There are 6 digits to choose from to get the three-digit
number. We must note that 0 cannot be in the first position, hence for
first position we restrict our choice to the other 5 digits. So the first digit
can be chosen in 5 ways; and for the second place we can now also choose
0, so we will have 4 remaining digits and a 0 hence five choices; and for
third position we have 4 ways. So we have

5 × 5 × 4 = 100 ways.

(b) With repetition; the first digit can be chosen in 5 ways excluding 0, but
second and third position we will have 6 choices since we are allowed to
repeat the digit already used and 0 is included here. So we have

5 × 6 × 6 = 180 ways.

Example 3.3.4. In a beauty pageant competition, how many ways can we


select a queen, first princess and second princess if there are 30 contestants who
entered the competition?

Solution:

This is just permuting 3 from 30 contestants hence

30 30! 30! 30 × 29 × 28 × 27!


P3 = = = = 30 × 29 × 28 = 24 360.
(30 − 3)! 27! 27!

Theorem 3.3.5. Given a set of n objects of which n1 are of one kind, n2


are of the second kind,· · · , and nk are of the k-th kind, then the number of
distinguishable permutations Pd is

n!
Pd = . (3.1)
n1 !n2 ! · · · nk !

84
Example 3.3.6. Given the word STATISTICS;

(a) How may distinct arrangements are there?

(b) How many distinct arrangements are there if the word start with letter
“S” and ends with “S”.

Solution:

(a) The word STATISTICS has 10 letters with 3 “S”, 3 “T”, 2 “I”, 1 “A” and
1 “C”.
Using equation 3.1, we get that the distinct arrangements are

10!
Pd = = 50 400.
3!3!2!

(b) Since we know what is in the first and last position, so in the case of 3 “S”,
we take out 2 and put them in first and last place and left with one. So we will
only permute the remaining 8 letters in between, hence

8!
Pd = = 3 360.
3!2!

3.3.2 Combination

Frequently we are not concerned with the order in which the objects are chosen,
but merely which objects are chosen, i.e. we may be concerned with a selection
or a combination.

Theorem 3.3.7. The number of r-combinations of a set with n elements, where


n is a nonnegative integer and r is an integer with 0 ≤ r ≤ n, equals

n n!
Cr = .
r!(nr)!

Example 3.3.8. How many different committees of four students can be formed
from a group of ten students?

85
Solution:

In this case , there is no specific order specified, so we are only interested in the
number of combinations without worrying about order.

10 10! 10!
C4 = = = 210.
4!(10 − 4)! 4!6!

Example 3.3.9. Suppose we have 5 boys and 7 girls. A team of 5 is to be


selected to play in a tournament. How many ways can a team be selected if,

(a) a team is randomly selected?

(b) a team chosen consists of 3 boys and 2 girls?

(c) a team chosen consists of more boys than girls?

Solution:

(a) In total there are twelve individual to choose from. So


12 12! 12!
C5 = 5!(12−5)! = 5!7! = 792.

(b) We must choose 3 boys from 5 and 2 girls from 7 in


5 5! 7! 5! 7!
C 3 ×7 C 2 = 3!(5−3)! × 2!(7−2)! = 3!2! × 2!5! = 10 × 21 = 210.

(c) For a team to have more boys than girls, we can have either have 3 boys
or 4 boys or 5 boys with 2 girls or 1 girl or 0 girls respectively, so

5
C3 ×7 C2 +5 C4 ×7 C1 +5 C5 ×7 C0
5! 7! 5! 7! 5! 7!
= × + × + ×
3!(5 − 3)! 2!(7 − 2)! 4!(5 − 4)! 1!(7 − 1)! 5!(5 − 5)! 0!(7 − 0)!
5! 7! 5! 7! 5! 7!
= × + × + ×
3!2! 2!5! 4!1! 1!6! 5!0! 0!7!
= 10 × 21 + 5 × 7 + 1 × 1

= 210 + 35 + 1 = 246.

Tutorials 3.3.10. Show your workings in each of the following questions:

1. Given the following digits, 0, 1, 3, 4, 7 and 8.

86
(a) How many three-digit odd integers can be formed without repetition?

(b) How many two-digit even integers can be formed with repetition?

(c) How many odd integers can be formed in total?

2. Using the the letters of English alphabets,

(a) How many seven letter words can be formed with and without repe-
tition?

(b) How many five letter words which consist of consonants only can be
formed without repetition?

(c) How many eight letter words can be formed which contains all vowels
of alphabets without repetition?

3. A committee of 5 people must be chosen such that there must be chairper-


son, vice-chairperson, secretary, vice-secretary and a treasurer. If there 15
people to choose from,

(a) How many ways can a committee be formed from this information?

(b) How many ways can a committee be formed if Mr Mabuza is a chair-


person?

(c) How many ways can a committee be formed if Mrs Jansen and Mr
Tlou are the ones occupying Chairperson and treasurer position?

4. Consider the word PHILLIPPINES. How many

(a) distinct arrangements can be formed considering the above letters?

(b) distinct arrangements such that the word starts and ends with P?

(c) start with L and ends with I.

(d) all the “P” goes together.

5. Six books lie on a table, and a girl is told that she can take away as many
as she likes but she must not leave empty handed.

(a) How many different selections can she make?

87
(b) One of these books is a Bible. How many of these selections will
include this Bible?

6. A bus needs 7 people to fill and in a bus stop there are 15 people waiting.

(a) How many possible outcomes are there to fill the bus?

(b) If Mary is one of the people in bus stop and she has to be inside the
bus, how many outcomes are there?

(c) Martin can only get inside the bus if and only if Mary gets inside.
How many outcomes are possible here?

“There is nothing so easy but that it becomes difficult when you do it reluctantly.”
Pythagoras

88
Chapter 4

COMPLEX NUMBERS

”To those who ask what the infinitely small quantity in mathematics is, we
answer that it is actually zero. Hence there are not so many mysteries hidden
in this concept as they are usually believed to be”. Leonhard Euler

4.1 Operations and Properties of Complex Num-


bers
In this chapter we are going to explore how complex numbers arose and the need
for them. We are also going to discuss the various ways of representing complex
numbers. We are going to learn how to perform various algebraic operations
with complex numbers, find roots of complex numbers and solve polynomial
equations.
Suppose we have to solve quadratic equation such as x2 + 1 = 0, i.e. we must
find x such that x2 = −1. In high school we learnt that there are no real roots
for such equation. In mathematics, it is possible to extend the number system
beyond the real numbers in order to solve equation such that x2 = −1. We call
these numbers imaginary numbers.

Definition 4.1.1. An imaginary number i is a number such that i2 = −1.

89
Definition 4.1.2. A complex number, is a number of the form a + bi where
a, b ∈ R and i2 = −1, where a is called the real part and b the imaginary
part.

The complex number z = a + bi is said to be in real-imaginary form. We write


x = Re(z) denoting x the real part of z and y = Im(z) denoting the imaginary
part of z. Note that the imaginary part of z is a real number.
√ √
Example 4.1.3. Let z = 5 − 3i and w = − 10 + 2i.
Solution:
√ √
Re(z) = 5; Im(z) = − 3 and Re(w) = − 10; Im(w) = 2.

4.1.1 Operations with complex numbers

For all the operations that follow below, we let z = a + bi and w = c + di.

Equality

The complex numbers z and w are said to be equal if and only if a = c and
b = d; i.e. z = w ⇔ a = c and b = d.

Addition and Subtraction

z + w = (a + bi) + (c + di)

= (a + c) + (b + d)i

z − w = (a + bi) − (c + di)

= (a − c) + (b − d)i

Example 4.1.4. If z = 7 − 2i and w = −4 + 7i, then write z + w and z − w in


real-imaginary form.

90
Then

z + w = (7 − 2i) + (−4 + 7i)

= (7 + (−4)) + (−2 + 7)i

= 3 + 5i

z − w = (7 − 2i) − (−4 + 7i)

= (7 − (−4)) + (−2 − 7)i

= 11 − 9i

Multiplication

zw = (a + bi)(c + di)

= ac + adi + bci + bdi2

= (ac − bd) + (ad + bc)i

Example 4.1.5. If z = −3 − 2i and w = 5 + i then write zw in real-imaginary


form.
zw = (−3 − 2i)(5 + i) = −15 − 3i − 10i − 2i2 = −15 + 2 − 13i = −13 − 13i.

Division

z a + bi
=
w c + di
(a + bi)(c − di)
=
(c + di)(c − di)
ac + bd + (bc − ad)i
=
c2 + d2
ac + bd bc − ad
+ 2 i.
c2 + d2 c + d2
z w
Example 4.1.6. If z = 5 − 2i and w = −6 − i then write w and z in real-
imaginary form.

91
z 5 − 2i
=
w −6 − i
(5 − 2i)(−6 + i)
=
(−6 − i)(−6 + i)
−30 + 5i + 12i − 2i2
=
62 + 12
−30 + 2 17 −28 17
= + i= + i.
37 37 37 37
w
z (exercise)

Example 4.1.7. Simplify the following expression and write the answer in
real-imaginary form.

(a) i(2 + 5i)2 ,

(2−i)(1+i)
(b) 4−3i ,

2
(c) i2 − i3 − i5 ,

Solutions:

(a)

i(2 + 5i)2 = i(4 + 20i + 25i2 )

= 4i + 20i2 − 25i

= −20 − 21i

92
(b)

(2 − i)(1 + i) 2 + 2i − i − i2
=
4 − 3i 4 − 3i
2+1+i
=
4 − 3i
3+i 4 + 3i
= ×
4 − 3i 4 + 3i
12 + 9i + 4i + 3i2
=
16 + 9
12 − 3 + 13i
=
25
9 13
= + i.
25 25

(c)

2 2
i2 − i3 − = −1 − ii2 − 2 2
i5 i(i )
2
= −1 + i −
i(−1)2
2 i
= −1 + i − ×
i i
2i
= −1 + i −
−1
= −1 + i + 2i

= −1 + 3i.

Example 4.1.8. Solve for x and y in the following expressions:

(a) 3(x − iy) = 5 − 9i,

(b) 3(x + i2y) − 2(x − iy) = 7 − 16i,



(c) 3 − i = x + iy,

(d) 3(2x + iy) − 2(y − ix) = 10 − 7i

93
Solutions:

(a)

3(x − iy) = 5 − 9i

3x − i3y = 5 − 9i

Equating the real part and imaginary parts on both sides we get
5
3x = 5 and −3y = −9 hence x = 3 and y = 3.

(b)

3(x + i2y) − 2(x − iy) = 7 − 16i

3x + i6y − 2x + i2y = 7 − 16i

x + i8y = 7 − 16i

Equating the real part and imaginary parts on both sides we get
x = 7 and 8y = −16 hence x = 7 and y = −2.

(c)
p
3−i= x + iy
p
(3 − i)2 = ( x + iy)2

9 − 6i + i2 = x + iy

9 − 1 − 6i = x + iy

8 − 6i = x + iy

Equating the real part and imaginary parts on both sides we get
x = 8 and y = −6.

94
(c)

3(2x + iy) − 2(y − ix) = 10 − 7i

6x + 3yi − 2y + 2xi = 10 − 7i

(6x − 2y) + (2x + 3y)i = 10 − 7i

Then,

6x − 2y = 10 (1)

2x + 3y = −7 (2)

(2) × 3 : 6x + 9y = −21 (1)


31
(1) − (2) : −11y = 31 ⇒ y = −
11
Substitute y = − 31
11 in (2), then

 31  63 14
2x + 3 − = −7 ⇒ 2x = −7⇒x=−
11 11 11
Theorem 4.1.9. Let z, w and w be complex numbers. Then

(a) z + w = w + z (addition is commutative)

(b) zw = wz (multiplication is commutative)

(c) z + (w + u) = (z + w) + u (addition is associative)

(d) z(wu) = (zw)u (multiplication is associative)

(e) z(w + u) = zw + zu or (z + w)u = zu + wu (distribution law)

(f ) z + 0 = 0 + z = z (addition by addition unit)

(g) z + (−z) = (−z) + z = 0 (additive inverse)

(h) z · 1 = 1 · z = z (multiplicative unit)

95
Tutorials 4.1.10. 1. Simplify the following and write in form a + ib (real-
imaginary form).

(a) (3 − i)[(2 + 3i)(4 − i)]

(b) (2 + 3i)[(−7 + i) − (2 − 3i)]

(c) 7(i − 3) − 3(2 + 3i)i


i(2+5i)
(d) 3−4i
h i
4
(e) (2i − 1)2 i−2 + 3+i
2+i
 3  2
1+i
(f) 4 i−1 − 3 1−i
1+i

i4 +i9 +i16
(g) 2−i5 +i10 −i15

2. Solve for x and y.

(a) 7(x − iy) = 21 − 7i,

(b) 2(2x − iy) + (2y − 3xi) = 5 − 3i,



(c) 4 − 3i = 2x + iy,

(d) 2(x − iy) − 3(xi − 3y) = 5 + i.

4.2 Modulus-Argument form (Polar form) of com-


plex numbers
We can also view the complex number z = a + ib as a ordered pair (a, b) of real
numbers. This representation facilitates the representation of complex numbers
as points in the xy-plane called the complex plane or Argand diagram.
Any complex number z represented in the the Argand plane is associated with
a unique distance from the origin measured in a direction defined by an angle
θ, made with the positive real-axis. This angle is not unique (why?).
The distance is called the modulus of z and the angle θ is called the argument
of z.

96
Definition 4.2.1. Consider the complex number z = a + ib. The modulus of
the complex number z, denoted by |z|, is the real number defined by
p
|z| = a2 + b2 .

The argument (arg) of z is defined to be the angle θ that the line from the
origin to makes with the real-axis.

Let z = a + ib and r = |z| = a2 + b2 . By using the polar coordinates (r, θ)
in the Argand diagram we can show that we can write

z = (a, b) = a + ib = r cos θ + ir sin θ = r(cos θ + i sin θ).

The form z = r(cos θ + i sin θ) is called the Modulus-Argument form (Polar


form) of complex number z.

Definition 4.2.2. Given complex number z = a + ib with r = |z| = a2 + b2 .
The unique real number θ such that a = r cos θ and b = r sin θ, −π < θ ≤ π, is
called the principal value of Argument of z, denoted by arg(z).

NB:

We must note that the Argument θ is only unique up to adding integer multiple
of 2π. Also note that
b
arg(a + ib) = arctan ,
a
provided a 6= 0.

Example 4.2.3. Find the modulus and Argument of the following complex
numbers and hence express each z in the Modulus-Argand form.

(a) z = 1 + i

(b) z = 1 − i

(c) z = −3 − 3i

(d) z = i

(e) z = −1

97
Solutions:

(a) z = 1 + i has ordered pair (1, 1) which tells us z is in the first quadrant in
the Argand diagram. Hence
1 π
θ = arg(z) = arctan = arctan 1 = .
1 4
√ √
Modulus: r = |z| = 12 + 12 = 2.

So z = r(cos θ + i sin θ) = 2(cos π4 + i sin π4 ).

(b) z = 1 − i has ordered pair (1, −1) which tells us z is in the fourth quadrant
in the Argand diagram. Hence
 −1  π
θ = arg(z) = arctan = arctan(−1) = − .
1 4
√ √
Modulus: r = |z| = 12 + 12 = 2.

So z = 2(cos π4 + i sin π4 ).

(c) z = −3 − 3i has ordered pair (−3, −3) which tells us z is in the third
quadrant in the Argand diagram. Hence
 −3  3π
θ = arg(z) = arctan = arctan(−1) = − .
−3 4
p √
Modulus: r = |z| = (−3)2 + (−3)2 = 18.
√ h    i
So z = 18 cos − 3π 4 + i sin − 3π
4 .

(d) z = i has ordered pair (0, 1) which tells us z is on the positive imarginary-
axis in the Argand diagram. Hence in this case we will find the general
solution where
cos θ = 0 and sin θ = 1.

The solution θ = π2 . Modulus: r = |z| = 02 + 12 = 1.
So z = cos π2 + i sin π2 .

(e) z = −1 has ordered pair (−1, 0) which tells us z is on the negative real-
axis in the Argand diagram. Hence in this case we will find the general
solution where
cos θ = −1 and sin θ = 0.

98
p
The solution θ = π. Modulus: r = |z| = (−1)2 + 02 = 1.
So z = cos π + i sin π.

We can summarize the argument value where z = a + ib by the following


table:

Position of z on the plane arg(z)


z is in first and fourth quadrant arctan ab
z is in second quadrant π + arctan ab
z is in third quadrant arctan ab − π
z is on positive real axis: b = 0, a > 0 0
π
z is on the positive imaginary axis: a = 0, b > 0 2

z is on negative real axis: b = 0, a < 0 π


z is on negative imaginary axis: a = 0, b < 0 − π2

Theorem 4.2.4. Suppose z = r1 (cos θ1 + i sin θ1 ) and w = r2 (cos θ1 + i sin θ2 ),


then
zw = r1 r2 [cos(θ1 + θ2 ) + i sin(θ1 + θ2 )],

and
z r1
= [cos(θ1 − θ2 ) + i sin(θ1 − θ2 )].
w r2
Definition 4.2.5. Let z = a + bi; then the conjugate of z, denoted by z is given
by:
z = a + bi = a − bi.

Example 4.2.6. Find the conjugate of z in each of the following:

(a) z = 5 + 2i

(b) z = −3 + i

(c) z = −3 − 3i

(d) z = 5i − 3

(e) z = −1

99
Solutions:

(a) z = 5 + 2i = 5 − 2i

(b) z = −3 + i = −3 − i

(c) z = −3 − 3i = −3 + 3i
√ √
(d) z = 5i − 3 = − 5i − 3

(e) z = −1 = −1.

Theorem 4.2.7. (Properties of conjugate) For all z, w ∈ C, then


 
(a) z + w = z̄ + w̄; z − w = z̄ − w̄; zw = z̄ w̄; wz = w̄z̄ ; and z̄¯ = z.

(b) z z̄ = |z|2 which is real and non-negative.

(c) z + z̄ = 2Re(z) and z − z̄ = 2Im(z)i.

(d) z ∈ R ⇒ z = z̄.

Theorem 4.2.8. For all z, w ∈ C, the modulus || function satisfies

(a) |z| ≥ 0; equality holding if z = 0.

(b) |zw| = |z||w|.



|z|
(c) wz = |w| if w 6= 0.

(d) Re(z) ≤ |Re(z)| ≤ |z|.

Theorem 4.2.9. (Inverse of a complex number). Let z ∈ C. If z 6= 0,


then there exist a unique z −1 ∈ C such that

zz −1 = 1.

We call z −1 a multiplicative inverse of z and can be represented as

z
z −1 = .
|z|2

100
Proof. If z = a + ib 6= 0, then either a 6= 0 or b 6= 0, hence |z|2 = a2 + b2 6= 0.
a b
Let w = a2 +b2 − a2 +b2 i. Then
 a b 
zw = (a + ib) − i
a 2 + b2 a2 + b2
2
a ab ab b2
= 2 2
− 2 2
i+ 2 2
− 2 i2
a +b a +b a +b a + b2
a2 b2
= 2 +
a + b2 a2 + b2
2 2
a +b
= 2
a + b2
= 1.

So z −1 = w = a
a2 +b2 − b
a2 +b2 i = a−ib
a2 +b2 = z
|z|2 . 

Tutorials 4.2.10. Find the Modulus and argument of the following complex
numbers and hence write in in polar form.

1. (a) z = 3+i

(b) z = 3i

(c) z = −2 + 2i

(d) z = 10

(e) −4 3 − 4

2. Find conjugate of the above complex numbers.

4.3 Euler’s Formula (Exponential Form)


We can also express complex numbers using Euler’s formula which involves

the exponent of power e. If z = a + ib, with r = |z| = a2 + b2 . Then we can
represent z by Euler’s formula as follows,

z = reiθ , where r = |z| = a2 + b2 and θ = arg(z).

It is important to note here that θ = arg(z) is in radians only in Euler’s formula.

101
Example 4.3.1. Write the following in Euler’s form.
h    i
(a) z = 5 cos π3 + i sin π3 .

(b) z = 13(cos 2 + i sin 2).

(c) z = 1 − i.

(d) z = −2 3 − 2i.

Solution:
h    i π
(a) z = 5 cos π3 + i sin π3 = 5e 3 i .

(b) z = 13(cos 2 + i sin 2) = 13e2i .


p √
(c) z = 1 − i. First r = |z| = 12 + (−1)2 = 2.
z is in fourth quadrant, so
−1 π
θ = arg(z) = arctan( ) = arctan(−1) = − .
1 4
√ π
So z = 2e− 4 i .
√ q √ √ √
(d) z = −2 3 − 2i. First r = |z| = (−2 3)2 + (−2)2 = 12 + 4 = 16 =
4.
z is in third quadrant, so
 −2   1 π 5π
θ = arg(z) = arctan √ − π = arctan √ ) − π = − π = − .
−2 3 3 6 6

So z = 4e 6 i .

Tutorials 4.3.2. Write the following in Euler’s form.



(a) z = 3+i

(b) z = 3i

(c) z = −2 + 2i

(d) z = 10

(e) −4 3 − 4i

102
4.4 De Moivre’s Theorem
Theorem 4.4.1. (De Moivre’s Theorem:)

(cos θ + i sin θ)n = cos nθ + i sin nθ for all n ∈ N.

Proof. Let P (n) : (cos θ + i sin θ)n = cos nθ + i sin nθ for all n ∈ N.
For n = 1: (cos θ + i sin θ)1 = cos 1 · θ + i sin 1 · θ = cos θ + i sin θ.
So P (1) is true.
Assume P (k) is true for some k ∈ N, i.e. (cos θ + i sin θ)k = cos kθ + i sin kθ.
We will now deduce the statement is true for n = k + 1; i.e.

(cos θ + i sin θ)k+1 = cos(k + 1)θ + i sin(k + 1)θ.

(cos θ + i sin θ)k+1 = (cos θ + i sin θ)k (cos θ + i sin θ)

= (cos kθ + i sin kθ)(cos θ + i sin θ) From assumption

= cos kθ cos θ + i cos kθ sin θ + i sin kθ cos θ + i2 sin kθ sin θ

= cos kθ cos θ − sin kθ sin θ + i(cos kθ sin θ + sin kθ cos θ)

= cos(kθ + θ) + i sin(kθ + θ)

= cos(k + 1)θ + i sin(k + 1)θ

So P (k + 1) is also true, so P (n) is true for all n ∈ N. 

Example 4.4.2. Use De Moivre’s Theorem to express the following in form


a + ib.

(a) (−2 − 2 3i)7
 12
(b) √1+i
3−i

Solutions:


(a) Let w = −2 − 2 3i.
q √ √ √
r = (−2)2 + (−2 3)2 = 4 + 12 = 16 = 4.

103
w is in third quadrant, so
 −2√3  √ π 2π
θ = arg(z) = arctan = arctan( 3) = − π = − .
−2 3 3
Then
√ n h  2π   2π io7
(−2 − 2 3i)7 = 4 cos − + i sin −
3 3
h  2π   2π i
7
= 4 cos − × 7 + i sin − ×7 De Moivre’s Theorem
3 3
h  14π   14π i
= 47 cos − + i sin −
3 3
 1 √3 
= 47 − −
2 2

= −8192 − 8192 3i
√ √
(b) Let z = 1 + i and w = 3 − i. |z| = 2 and |w| = 2.
π
arg(z) = and arg(w) =
4 − π6 .
√ h    i
 1+i  2 cos π4 + i sin π4
√ = h    i
3−i 2 cos − π6 + i sin − π6
1 h π π  π π i
= √ cos + + i sin + Using Theorem 4.2.4
2 4 6 4 6
1  5π 5π 
= √ cos + i sin
2 12 12

Now
 1+i 
12
h 1  5π 5π i12
√ = √ cos + i sin
3−i 2 12 12
1h  5π   5π i
= 6 cos × 12 + i sin × 12
2 12 12
1
= (cos 5π + i sin 5π)
64
1
= (−1 + 0)
64
1
=−
64

Example 4.4.3. Use De Moivre’s Theorem to show that:

(a) sin 4θ = 2 sin 2θ cos 2θ.

cos 5θ
(b) cos θ = 16 cos4 θ − 20 cos2 θ + 5.

104
Solutions:

(a)
4  
4
X 4
(cos θ + i sin θ) = (cos θ)4−r (i sin θ)r
r=0
r
   
4 4
= cos4 θ + (cos θ)3 (i sin θ) + (cos θ)2 (i sin θ)2
1 2
 
4
+ (cos θ)(i sin θ)3 + (i sin θ)4
3
= cos4 θ + 4i cos3 θ sin θ − 6 cos2 θ sin2 θ − 4i cos θ sin3 θ + sin4 θ

Since by De Moivre’s theorem, (cos θ + i sin θ)4 = cos 4θ + i sin 4θ, then

cos 4θ + i sin 4θ = (cos4 θ − 6 cos2 θ sin2 θ + sin4 θ) + i(4 cos3 θ sin θ − 4 cos θ sin3 θ).

So equating imaginary parts we get:

sin 4θ = 4 cos3 θ sin θ − 4 cos θ sin3 θ

= 4 cos θ sin θ(cos2 θ − sin2 θ)

= 2(2 cos θ sin θ)(cos 2θ)

= 2 sin 2θ cos 2θ

(b)

5  
X 5
(cos θ + i sin θ)5 = (cos θ)5−r (i sin θ)r
r=0
r
   
5 5
= cos5 θ + (cos θ)4 (i sin θ) + (cos θ)3 (i sin θ)2
1 2
   
5 5
+ (cos θ)2 (i sin θ)3 + (cos θ)(i sin θ)4 + (i sin θ)5
3 4
= cos5 θ + 5i cos4 θ sin θ − 10 cos3 θ sin2 θ − 10i cos2 θ sin3 θ

+ 5 cos θ sin4 θ − i sin5 θ

cos 5θ + i sin 5θ = (cos5 θ − 10 cos3 θ sin2 θ + 5 cos θ sin4 θ)

+ i(5 cos4 θ sin θ − 10 cos2 θ sin3 θ − sin5 θ).

105
Taking the real parts

cos 5θ = cos5 θ − 10 cos3 θ sin2 θ + 5 cos θ sin4 θ


cos 5θ
= cos4 θ − 10 cos2 θ sin2 θ + 5(sin2 θ)2
cos θ
= cos4 θ − 10 cos2 θ sin2 θ + 5(1 − cos2 θ)2

= cos4 θ − 10 cos2 θ(1 − cos2 θ) + 5(1 − 2 cos2 + cos4 θ)

= cos4 θ − 10 cos2 θ + 10 cos4 θ + 5 − 10 cos2 +5 cos4 θ

= 16 cos4 θ − 20 cos2 θ + 5.

Tutorials 4.4.4. 1. Use De Moivre’s Theorem to write the following in form


a + ib.

(a) z = (4 − 4 3i)8
 √ 
3+i 9
(b) − √
1+ 3i

(1− 3i)3 (1−i)8
(c) 5
(−2+2i

2. Use De Moivre’s Theorem to prove that:

(a) sin3 θ = 3
4 sin θ − 1
4 sin 3θ.
1 1
(b) cos4 θ = 8 cos 4θ + 2 cos 2θ + 83 .

4.5 Finding roots of Complex Numbers


Theorem 4.5.1. If z 6= 0, n ∈ N, then there are exactly n distinct complex nth
roots of z .i.e.there are n distinct complex numbers z1 , z2 , . . . , zn with zkn = z,
k = 0, 1, 2, . . . , n − 1:Furthermore,

1
h  arg(z) + 2πk   arg(z) + 2πk i
zk = |z| n cos +i sin for k = 0, 1, 2, . . . , n − 1,
n n

Remarks 4.5.2. In the Argand diagram the n nth roots of z are equally spaced
out on a circle of radius |z| = r.
h    i
Example 4.5.3. (a) Find the cube root of z = 27 cos π3 + i sin π3 .

106
(b) Solve z 5 + 32 = 0 i.e. find the 5th root of −32.
√ √
(c) Solve z 6 − 1 = − 3i i.e. find the 6th root of 1 − 3.

Solutions:
h π  π i
1 +2πk +2πk
(a) zk = (27) 3 cos 3 n + i sin 3 n for k = 0, 1, 2.

1
h  π + 2π0i   π + 2π0i i
z0 = (27) 3 cos 3 + i sin 3
3 3
h π  π i
= 3 cos + i sin
9 9
π π
= 3 cos + i3 sin
9 9
= 2.82 + 1.03i

also

1
h  π + 2π   π + 2π i
z1 = (27) 3 cos 3 + i sin 3
3 3
h  7π   7π i
= 3 cos + i sin
9 9
 7π   7π 
= 3 cos + i3 sin
9 9
= −2.30 + 1.93i

finally

1
h  π + 2π(2)   π + 2π(2) i
z2 = (27) 3 cos 3 + i sin 3
3 3
h  13π   13π i
= 3 cos + i sin
9 9
 13π   13π 
= 3 cos + i3 sin
9 9
= −0.52 − 2.95i

(a) z 5 + 32 = 0 ⇒ z 5 = −32 = 32(−1 + 0i) = 32(cos π + i sin π) then

1
h  π + 2πk   π + 2πk i
zk = (32) 5 cos + i sin for k = 0, 1, 2, 3, 4.
5 5

107
h π  π i
z0 = 2 cos + i sin
5 5
π π
= 2 cos + i2 sin
5 5

also
h  π + 2π   π + 2π i
z1 = 2 cos + i sin
5 5
 3π   3π 
= 2 cos + i2 sin
5 5

h  π + 4π   π + 4π i
z2 = 2 cos + i sin
5 5
= 2 cos π + i2 sin π

= −2

h  π + 6π   π + 6π i
z3 = 2 cos + i sin
5 5
 7π   7π 
= 2 cos + i2 sin
5 5

h  π + 8π   π + 8π i
z4 = 2 cos + i sin
5 5
 9π   9π 
= 2 cos + i2 sin
5 5

√ √  √
3

(c) z 6 − 1 = − 3i ⇒ z 6 = 1 − 3i = 2 12 − 2 i
h    i
= 2 cos − π3 + i sin − π3 then

1
h  − π + 2πk   − π + 2πk i
3 3
zk = (2) 6 cos + i sin for k = 0, 1, 2, 3, 4, 5.
6 6

108
1
h −π + 0  − π + 0 i
3 3
z0 = (2) 6 cos + i sin
6 6
1
h  π  π i
= (2) 6 cos − + i sin −
18 18
1
 π  1
π
= (2) 6 cos − i(2) 6 sin
18 18

also
1
h  − π + 2π   − π + 2π i
3 3
z1 = (2) 6 cos + i sin
6 6
1
 5π  1
 5π 
= (2) cos
6 + i(2) sin
6
18 18

1
h  − π + 4π   − π + 4π i
3 3
z2 = (2) 6 cos + i sin
6 6
1
 11π  1
 11π 
= (2) 6 cos + i(2) 6 sin
18 18

1
h  − π + 6π   − π + 6π i
3 3
z3 = (2) 6 cos + i sin
6 6
1
 17π  1
 17π 
= (2) 6 cos + i(2) 6 sin
18 18

1
h  − π + 8π   − π + 8π i
3 3
z4 = (2) 6 cos + i sin
6 6
1
 23π  1
 23π 
= (2) 6 cos + i(2) 6 sin
18 18

1
h  − π + 10π   − π + 10π i
3 3
z5 = (2) 6 cos + i sin
6 6
1
 29π  1
 29π 
= (2) 6 cos + i(2) 6 sin
18 18

Tutorials 4.5.4. Find the roots of:

109
(a) z 4 = 1,

(b) z 2 = −1 + i

(c) z 4 = −8 + 8 3i

(d) z 5 = −2 2i

(e) z 6 = −4 3 − 4i

110
Chapter 5

INTRODUCTION TO
MATRICES AND LINEAR
EQUATIONS

5.1 Matrix Theory


In this section we investigate about the concept of matrix and some of it’s
properties.
We will denote a matrix by capital letters A, B, C, D,etc.

Definition 5.1.1. A matrix A is a rectangular array of numbers represented


in the form:  
a11 a12 ... a1n
 
 
 a21 a22 . . . a2n 
 
 
. . . . . . . . . . . . . . . . . . . . .
 
am1 am2 . . . amn
The number aij is called an entry or member, which is in row i and column j
of the matrix. A matrix with m rows and n columns is called an m by n matrix,

111
written m × n. The pair of numbers m and n is called the order or size of the
matrix.

Some Important Matrix Properties:

• A square matrix is one which has the same number of rows as columns
i.e. m × n matrix with (m
= n). 
  1 3 −1
0 8  
e.g. A =   B = 4 −7 1  where A is 2 × 2 and B is 3 × 3
 
12 −1  
1 2 −5
matrices respectively.

• A matrix with all entries equal to zero is called a zero matrix, and is
denoted by 0m×n ; 
0 0 0
e.g. 02×3 =  
0 0 0

• An identity matrix denoted by In , is a square matrix with aii = 1 for


all i and zero elsewhere;  
  1 0 0
1 0  
e.g. I2 = and I3 = 0
 
  1 0
0 1  
0 0 1

• A matrix with one row (m=1) is called a row matrix;


 
A = −8 5 4 2

• A matrix
  with one column (n=1) is called a column matrix;
1
 
B = −7
 
 
8

• Two 
matrices are equal
 if 
their sizes andcorresponding
 entriesare equal;
0 8 12 1 3 −1 0 8 12
A= ; B =  ; C =  , then
−2 5 −1 4 −7 1 −2 5 −1

A 6= B; B 6= C; A = C.

112
• For an m × n matrix A = (aij ), the transpose of A is the n × m denoted
by At = (aji ) which is obtained by interchanging the row and columns of
A;  
5 8  
  5 −2 7
A = −2 −1 then At =  .
 
  8 −1 0
7 0
Note: If A is a m × n matrix, then At is an n × m matrix.

• A diagonal matrix is one with at least one non-zero entry in the leading
(main) diagonal and zeroes everywhere;
 
9 0 0
 
A = 0 0 0 
 
 
0 0 −2

• A matrix
 A is symmetric
 if At =A; 
5 8 −3 5 8 −3
   
t
A =  8 −1 −4 then A =  8 −1 −4.
   
   
−3 −4 0 −3 −4 0

• Scalar multiple: Given a matrix A and a real number k, then the matrix
kA is a matrix obtained by multiplying each entry of matrix A by k. The
matrix kA is called a matrix multiple of A. When dealing with matrices,
real numbers are often called scalars hence the scalar multiplication
for multiplying A by k.
 
2 −3 3
E.g.: If A =  , then
0 −1 5
 
−6 9 −9
−3A =  
0 3 −15

and  
1 1 − 23 3
2
A=
2 0 −1 5
2 2

113
5.1.1 Matrix Addition and Subtraction

We can only add or subtract two matrices if they have the same order. The
sum or difference of two matrices is obtained by adding or subtracting the
corresponding entries respectively. If
   
a11 a12 a13 b b12 b13
A=  , B =  11 ,
a21 a22 a23 b21 b22 b23

then
     
a11 a12 a13 b b12 b13 a + b11 a12 + b12 a13 + b13
A+B =  + 11  =  11 
a21 a22 a23 b21 b22 b23 a21 + b21 a22 + b22 a23 + b23

Similarly for subtraction


     
a11 a12 a13 b b12 b13 a − b11 a12 − b12 a13 − b13
A−B =  − 11  =  11 
a21 a22 a23 b21 b22 b23 a21 − b21 a22 − b22 a23 − b23

Example 5.1.2.
   
−1 8   0 −1  
  1 3   9 0
A= 5 0 ;B =  ;C =  0 ;D =  .
  
1
  4 −7   −3 −6
2 −3 −2 −5

       
−1 8 0 −1 −1 + 0 8 + (−1) −1 7
       
A+C =  5 0 + 1 0 = 5+1 0+0 = 6 0 .
       
       
2 −3 −2 −5 2 + (−2) −3 + (−5) 0 −8

       
1 3 9 0 1−9 −8 3−0 3
B−D = − = = .
4 −7 −3 −6 4 − (−3) −7 − 6 7 −13

A ± B; A ± D; B ± C or C ± D do not exist.

5.1.2 Matrix Multiplication

Consider m × n matrix A = (aij ) and p × q matrix B = (bij ). Then the matrix


multiplication AB is defined if and only if n = p. The product AB will be m × q

114
matrix whose ij-entry is obtained by multiplying the ith row of A by the jth
column of B. i.e.

    
a11 a12 ... a1n b11 b12 ... b1q c11 a12 ... a1q
    
    
 a21 a22 . . . a2n   b21 b22 . . . b2q   a21 a22 . . . a2q 
  = 
    
 . . . . . . . . . . . . . . . . . . . . .  . . . . . . . . . . . . . . . . . . . . . . . . . . . . . . . . . . . . . . . .
    
am1 am2 . . . amn bn1 bn2 . . . bnq am1 am2 . . . amq
Pn
Where cij = ai1 b1j + ai2 b2j + . . . + ain bnj = k=1 aik bkj .

Note 5.1.3. • If AB exist, it does not mean that BA also exist.

• Also in general matrix multiplication is not commutative, i.e. AB 6= BA


(although in some cases it is possible).

• If both AB and BA exist, they may not be of the same order.

Example 5.1.4.
 
  −3 4 0
−1 −3 5  
A=  2 × 3 matrix; B = 
1 5 −1 3 × 3 matrix.

2 0 −4  
2 0 3

AB exist since number of column in A is same as number of rows in B.


 
  −3 4 0
−1 −3 5  
AB =    1 5 −1

2 0 −4  
2 0 3
 
(−1)(−3) + (−3)(1) + (5)(2) (−1)(4) + (−3)(5) + 5(0) (−1)(0) + (−3)(−1) + 5(3)
= 
2(−3) + 0(1) + (−4)(2) 2(4) + 0(5) + (−4)(0) 2(0) + 0(−1) + (4)(−3)
 
10 −19 18
= 
−14 8 −12

Result of AB is 2 × 3 matrix. BA does not exist since the number of columns


in B is not equal to the number of rows in A.

115
Verify that BAt exists and
 
−9
−6
 
BAt = −21 6  .
 
 
13 −8

Remarks 5.1.5. If A is a matrix. Then An where n ∈ N, is a matrix obtained


by multiplying the matrix A by itself n times, e.g. A3 = AAA.
Check that this only occurs when A is a square matrix.

Some Properties of Matrices

(i) A + B = B + A

(ii) (A + B) + C = A + (B + C)

(iii) A(B + C) = AB + AC

(iv) (AB)C = A(BC)

(v) k(A + B) = kA + kB

(vi) (A + B)t = At + B t

(vii) (AB)t = B t At

(viii) AB 6= BA

Tutorials 5.1.6. 1. Consider


 the following
 matrices:  
  −2 7   2 −1 5
2 −3 1   3 −3  
A= ;B= 2 0 ; C = ; D = 0 1 0;
       
5 −1 0   0 −6  
3 −4 3 −1 5
   
2 0 3 5 0 0
     
E = −1 1 −1; F = 0 −3 0; G = 4 1 −2 ;
   
   
5 0 5 0 0 2
 
0
H =  ;
−3

116
(a) Which of the matrices above are: (i) Diagonal matrix; (ii) Square
matrix; (iii) Symmetric; (iv) Identity matrix; (v) Zero Matrix (vi)
Column Matrix; (vi) Row matrix.

(b) Which of the following matrices is matrix addition, subtraction and


multiplication possible?

(c) Which of the following operations are possible? Where the opera-
tions are possible evaluate: AC; B 3 ; C t F ; DEB; At C − B; (3E t )B;
(−3I3 − D)t ; HGB CGt .

(d) Write down the following entries: (i) b22 in B; (ii) e32 in E; (iii) a31
in A; (iv) f23 in F

2. Solve
 for a, b, c and 
d given
 that     
2a − b 3c − d 7 −3 a 3b 3 b−3
(i)   =   (i) 5  +  =
c + d a + 4b 0 −4 2c −d 0 −4
 
2a b−a
 
3c + d 7d

3. If A is a 5 × 4 matrices and AI = A, what is the size of I, the identity


matrix?

4. If AB = 0 and A is a non-zero matrix, can we conclude that B = 0?


 
x 2x xy
 
5. Determine x and y such that the given matrix is symmetric: A = y −1 x  .
 
 
y x2 x

5.2 Echelon Matrices


• Row equivalence matrices: Let the ith row of a matrix A be denoted
by Ri . The following are called elementary row operations:

1. Inter-change any two rows, Ri ↔ Rj

2. Multiply a row by a non-zero constant k, Ri → kRi

3. Add a multiple of any row to another row, Ri → Ri + kRj .

117
• Let A and B be m × n matrices. Then A is said to be row equivalent
to B if B can be obtained from A by a finite sequence of elementary row
operations.

We shall see how to apply this from next examples as well throughout the
chapter.

Definition 5.2.1. A matrix is said to be in row echelon form if it satisfies


then following properties:

(a) Every non-zero row begin with a leading one.

(b) A leading one in a lower row is further to the right.

(c) Zero rows are at the bottom of the matrix.

Example
 5.2.2. The following
 matrices are in
 a row echelon form:
1 0 −2 3 1 8 −2 7 2
   
0 1 −1 −4
   
0 0 1 3 1
(a) 

 (b) 
 
.

0 0 1 0 0 0 0 1 1
   
0 0 0 1 0 0 0 0 0

Definition 5.2.3. A matrix is said to be in reduced row echelon form if it


is a row echelon form and furthermore it has additional property that:

• every column with a leading one has zeros elsewhere.

Example
 5.2.4. The following
 matricesare in a reduced row echelon form:
1 0 0 0 1 0 0 0 3
   
   
0 1 0 0 0 0 1 0 0
(a) 

 (b) 
  .

0 0 1 0 0 0 0 1 2
   
0 0 0 1 0 0 0 0 0

Example 5.2.5. Reduce the following matrix and leave your answer in

(a) Row echelon form.

(b) Reduced row echelon form.

118
 
1 1 1 1 1
3
 
 
2 1 1 1 2 4
 .
−1 −1
 
1 1 1 5
 
1 0 0 1 1 4

Solution: (a)
 
1 1 1 3 1 1
 
 
2 1 1 1 2 4
 
−1 −1 1 1 5
 
1
 
1 0 0 1 1 4
 
1 1 1 1 1 3
 
0 −1 −1 −1 0 −2 R2 − 2R1
 
∼
 
0 −2 −2 0 0 2  R3 − R1

 
0 −1 −1 0 0 1 R4 − R1
 
1 1 1 1 1 3
 
1 1 0 2 −R2
 
0 1
∼ 
0 −2 −2 0 0 2

 
0 −1 −1 0 0 1
 
1 1 1 1 1 3
 
 
0 1 1 1 0 2
∼


0 0 0 2 0 6 R3 + 2R2
 
0 0 0 1 0 3 R4 + R2
 
1 1 1 1 1 3
 
 
 0 1 1 1 0 2
∼ 
0 0 0 1 0 3 21 × R3
 
 
0 0 0 1 0 3
 
1 1 1 1 1 3
 
 
0 1 1 1 0 2
∼



0 0 0 1 0 3
 
0 0 0 0 0 0 R4 − R3

119
This matrix is now in a row echelon form.
(b) Now to obtain a reduced row echelon form, we continue from the last step in
(a) to ensure that all columns with leading one have zeros elsewhere as follows:
 
1 1 1 0 1 0 R1 − R3
 
0 −1 R2 − R3
 
0 1 1 0
 
 
0 0 0 1 0 3
 
0 0 0 0 0 0
 
1 0 0 0 1 0 R1 − R2
 
0 0 −1
 
0 1 1
∼ 


0 0 0 1 0 3
 
0 0 0 0 0 0

Tutorials 5.2.6. Reduce the following matrix and leave your answer in

(a) Row echelon form.

(b) Reduced row echelon form.


 
1 2 −3 −2 4 1
 
1. 2 5 −8 −1 6 4 .
 
 
1 4 7 5 2 8
 
1 2 1 2 1 2
 
 
2 4 3 5 5 7
2. 
 .

3 6 4 9 10 11
 
1 2 4 3 6 9

5.3 Determinant
Given any square matrix A, there is a special scalar called the determinant of
A denoted by det(A) or we make use of vertical bars |A|. This number is also
important in determining whether a matrix is invertible or not, which will be
studied in the next section.
We will show how to evaluate determinant of square matrices of different orders.

120
(a): 1 × 1 matrix:

If A = (a), then we simply say det A = a.

Example 5.3.1. A = (2) gives det A = 2.

(b): 2 × 2 matrix:
 
a b
If A =  ;
c d


a b
The determinant is defined by: det A = = ad − bc

c d
 
5 −1
Example 5.3.2. If A =  ,then
6 −3

−1

5
det A = = (5)(−3) − 6(−1) = −15 + 6 = −9.
−3

6

Definition 5.3.3. The (i, j)-th minor of a matrix A, denoted by mij (A), is the
determinantobtained by deleting
 row i and column j of A.
a a12 a13
 11 


a11 a13
e.g. If A = a21 a22 a23 , then m32 (A) = .
 

  a21 a23
a31 a32 a33

Definition 5.3.4. The (i, j)-th cofactor of a matrix A, denoted by cij (A), is
the signed minor
 given by cij(A) = (−1)i+j mij (A) of A.
a a12 a13
 11 
e.g. If A = a21 a22 a23 , then
 
 
a31 a32 a33


a11 a12 a a12
c23 (A) = (−1)2+3 = − 11 .

a31 a32 a31 a32

121
(c): 3 × 3 matrix:

(i) Laplace expansion:


 
a a12 a13
 11 
Given A = a21 a22 a23 ;
 
 
a31 a32 a33



a11 a12 a13

det A = a21

a22 a23


a31 a32 a33

= a11 (−1)1+1 m11 + a12 (−1)1+2 m12 + a13 (−1)1+3 m13




a
2 22
a23 a
3 21
a23 a
4 21
a22
= a11 (−1) + a12 (−1)

+ a13 (−1)
.

a32 a33 a31 a33 a31 a32

This is Laplace expansion along row one. This expansion can be done
along any other row or column without affecting the value of the determi-
nant.
 
−2 3 −1
 
Example 5.3.5. A=  4 −7
 
1
 
1 2 −5

−2 −1

3

det A = 4 −7

1

−5

1 2

Expand along row 1



−7 −7

1+1
1 4 1 4
+ 3(−1)1+2 + (−1)(−1)1+3

= (−2)(−1)
−5 −5

2 1 1 2

= (−2)(35 − 2) − 3(−20 − 1) − 1(8 + 7) = −66 + 63 − 15 = −18.

(ii) Sarrus scheme:

122
Another useful method which can be used to evaluate the determinant of a
3 × 3 matrix is what we call “Sarrus scheme” or also commonly referred to as
“butterfly method ”. To find the det(A) using this method we follow the following
steps:

(i) Rewrite the 1st and 2nd columns on the right (as Columns 4 and 5).

(ii) Add the products along the three full diagonals that extend from upper
left to lower right.

(iii) Subtract the products along the three full diagonals that extend from
lower left to upper right.

Let’s work the previous example, now using “Sarrus scheme”:


E.g. Find
−2 −1

3

−7

4 1

−5

1 2
Solution:

−2 −1

3

det A = 4 −7

1

−5

1 2

−2 −1 −2 3

3

= 4 −7 1 4 −7


−5 1

1 2 2

= (−2)(−7)(−5) + (3)(1)(1) + (−1)(4)(2) − (1)(−7)(−1) − (2)(1)(−2) − (−5)(4)(3)

= −70 + 3 − 8 − 7 + 4 + 60 = −18.

NB:Sarrus rule does not work for n × n matrices where n ≥ 4.

Properties of determinants:

1. The sign of a determinant changes if any two columns or rows are inter-
changed.

123
E.g. From an example, we saw that

−2 3 −1


4 −7 1 = −18.


2 −5

1

If we interchange row 1 and row 3, the new determinant;



2 −5

1

4 −7 1 = 18.


−2 3 −1

Interchanging of columns has the same effect.

2. The determinant
of
a matrix with a zero column or zero row is zero.
1 0 −5


1 2
E.g. 4 0 1 = 0 and = 0.


0 0
−2 0 −1

3. If any two rows or columns of a matrix are equal, its determinant


is zero.

1 2 4 1
1 2 −5

−3 −5 −3

5
E.g. 8 −5 −2 = 0 since R2 = R3 . = 0 since

−4 −5 −4

0
8 −5 −2

1 −5 2

2
C1 = C4 .

4. If any row or column of a matrix is multiplied by a non-zero constant k,


it’s determinant is multiplied by k. E.g.If
 
−2 3 −1
 
A =  4 −7 1  ,
 
 
1 2 −5

then if we multiply column 3 by -5, i.e. we have new matrix


 
−2 3 5
 
B =  4 −7 −5 ,
 
 
1 2 25

124
then det B = −5 det A = −5(−18) = 90.
Suppose we multiply the whole matrix A by 2, then each row will be
multiplied by two. So

det 2A = 2 × 2 × 2 det A = 8 × (−18) = −144.

This is because each row or column is a multiplied by 2, hence the multiple


of each row multiplies the det A, in this case three 2’s since we have 3 rows
or columns.

In general If A is an n × n matrix, then det(kA) = k n det A.

5. If any row or column of a matrix is multiplied by a non-zero constant k


and the result is added to another row or column, it’s determinant remains
the same. We can show this by performing row operation.
E.g.  
−2 3 −1
 
A= 4 −7
 
1
 
1 2 −5
using row operation.

−2 3 −1 0 −11 R1 + 2R3

7

det A = 4 −7 1 = 0 −15 21 R2 − 4R3


2 −5 1 −5 −2

1

Expanding along C1

−11

7
=1×

−15 21

= 7 × 21 − (−15)(−11) = 147 − 165 = −18.

Which still yields the same result.


NB:Care must be taken when performing row operation as to which row
we multiply.
Performing row operation to find the determinant is quite helpful when

125
we deal with higher order square matrices, e.g. 4 × 4, 5 × 5, etc.
E.g. Consider a 4 × 4 matrix
 
1 3 5 7
 
 
3 1 3 5
D=



5 3 1 3
 
7 5 3 1



1 3 5 7 1 3 5 7

−8 −12 −16 R2 − 3R1

3 1 3 5 0
=
−12 −24 −32 R3 − 5R1

5 3 1 3 0

−16 −32 −48 R4 − 7R1

7 5 3 1 0

−8 −12 −16


= 1 × −12 −24 −32


−16 −32 −48



2 3 4

= (−4)(−4)(−16) 3 6 8



1 2 3

0 −1 −2 R1 − 2R3


= −256 0 0 −1 R2 − 3R3



1 2 3

−1 −2

= −256 × 1 ×
0 −1

= −256 × (1 − 0) = −256.

126

a2 a a2

a b+c a+b+c


b 2 = 2
c+a b b a+b+c b

2
c2

c a+b c c a+b+c

C2 + C1

a 1 a2


= (a + b + c) b 1 b2


c 1 c2


a2

a 1

= (a + b + c) b − a 0 b2 − a2 R2 − R1


c − a 0 c2 − a2 R3 − 1R1

Expand along C2

b − a (b − a)(b + a)

= −(a + b + c)
c − a (c − a)(c + a)



1 b + a
= −(a + b + c)(b − a)(c − a)

1 c + a

= −(a + b + c)(b − a)(c − a)(c + a − b − a)

= −(a + b + c)(b − a)(c − a)(c − b)

6. det(AB) = det A det B

7. det(A + B) 6= det A + det B.

8. det(A) = det(At )

Tutorials 5.3.6. 1. Find the value of x such that:



x − 3

5
(a) = 45
−3 x − 2


x −1 2x


(b) 1 −1 0 = 12


2 −1 −x

127
2. Use properties of determinants to evaluate:

3 −4 −2


(a) 6 −4 −1



9 2 3

1 k k 2


(b) 1 k k 2


1 k k 2



3 3 0 5

0 −2

2 2
(c)

4 1 −3 0



2 10 3 2


4 0 0 1 0

3 3 −1 0

3

(d) 1

2 4 2 3


2 2 4 2 3

−3 −1 2 0 9

3. Show that the value of the following determinant is independent of x.




sin x cos x 0

− cos x 0 = 0.

sin x

sin x − cos x sin x + cos x 1

5.4 Matrix Inversion


Definition 5.4.1. An n×n square matrix A is invertible (non-singular) if there
exist a matrix D such that AD = DA = In . We say D is an inverse matrix of
A and D = A−1 .

Note 5.4.2. The following facts are important:

• If A−1 exists, then it is unique and square of the same order as A.

128
• An invertible matrix and its inverse commute with respect to matrix mul-
tiplication i.e. AA−1 = A−1 A = In .

• A non-square matrix can never be invertible, and also among square ma-
trices there are infinitely many that are not invertible.

• There is a symmetric relationship between a matrix and it’s inverse; i.e.


if B is the inverse of A, then A is also an inverse of B.

• If A−1 exists, then the system of linear equations with A as a coefficient


matrix ALWAYS has a unique solution, whatever the right hand side.
We can actually write the equation as AX = B where X is the column
matrix of variables x, y, z, etc and B is the column matrix of the right
hand side. Then AX = B implies A−1 AX = A−1 B, then In X = A−1 B,
hence X = A−1 B. This will yield the unique solution. (We’ll touch this
in detail later).

We will study two methods of finding the inverse of a matrix. The first method
involves Gauss-Jordan operation and the second is the adjoint method.

A. Gauss-Jordan elimination method:


We will make use of the following algorithm called Matrix inversion algo-
rithm in order to evaluate the inverse using the Gauss-Jordan elimination.

NB: Matrix Inversion Algorithm:


If A is a (square) invertible matrix, there exists a sequence of elementary
row operations that carry A to the identity matrix I of the same size,
written A → I. This same series of row operations carries I to A−1 ; that
is, I → A−1 . The algorithm can be summarized as follows:

[A|I] → [I|A−1 ]

where the row operations on A and I are carried out simultaneously.

Example 5.4.3. Use the Gauss-Jordan elimination to find the inverse of

129
the following matrices:
 
  2 7 1
−2 3  
(a) A =   ; (b) B =  −1

1 4
4 −5  
1 3 0

(a)
 
−2 3 1 0

(A|I) =  
4 −5 0 1

 
−2 3 1 0

=


0 1 2 1 R2 + 2R1
 
−2 0 −5 −3 R1 − 3R2

=


0 1 2 1
 
1 0 25 32 (− 12 ) × R1

=


0 1 2 1

 
5 3
∴ A−1 = 2 2
.
2 1
(b)
 

2 7 1 1 0 0
 
(A|I) = 1 −1 0
 
4 1 0
 

1 3 0 0 0 1

130
First interchange row 1 and 2.
 
−1 0 1 0 R2 ↔ R1

1 4
 
∼ 2
 
7 1 1 0 0
 

1 3 0 0 0 1
 
4 −1 0 1 0

1
 
∼ 0 −1 3 1 −2 0 R2 − 2R1
 
 
−1 1 1 −1 1 R3 − R1

0
 
4 −1 0

1 1 0
 
∼ 0 1 −3 −1 2 0 −R2
 
 
−1 1 0 −1 1

0
 
0 11 4 −7 0 R1 − 4R2

1
 
∼ 0 1 −3 −1 2 0
 
 
0 −2 −1 1 1 R3 + R2

0
 
0 11 4 −7

1 0
 
∼ 0 1 −3 −1 2
 
0 
 
0 1 21 − 12 − 12 − 12 R3

0
 
3 3 11
1 0 11 − 2 − 2
2  R1 − 11R3

1 1
∼ 0 1 −3 2 − 32  R2 + 3R3
 
 2 
0 1 12 − 12 − 21

0

   
−3 −3 11
−3 −3 11
 2 2 2   
Hence A−1 =  1 1 −3  = 1
−3
  
2 2 2 
 2 1 1
  
1 −1 −1
2 2 2 1 −1 −1

B. The Adjoint Method:

Definition 5.4.4. The adjoint of a matrix A, denoted by adjA, is the


transpose of the matrix of the cofactors of A.

131
For example if  
a a12 a13
 11 
A = a21 a23  ;
 
a22
 
a31 a32 a33
then the matrix of cofactors of A, which we will denote by C is:
 
c11 (A) c12 (A) c13 (A)
 
C = c21 (A) c22 (A) c23 (A) ,
 
 
c31 (A) c32 (A) c33 (A)
then the adjoint matrix
 
c11 (A) c21 (A) c31 (A)
 
adjA = C T = c12 (A) c22 (A) c32 (A) ,
 
 
c13 (A) c23 (A) c33 (A)
Definition 5.4.5. If det A 6= 0 then the inverse of A is given by
1
A−1 = (adjA).
det A
Let again find the inverse of
 
2 7 1
 
B = 1 −1
 
4
 
1 3 0
using the adjoint method to verify that we will get the same answer.
To find the matrix of cofactors,
 
4 −1 1 −1 1 4








 3 0 1 0 1 3 
 
 
 7 1 2 1 2 7 
C= −










 3 0 1 0 1 3 
 
 
 7 1 2 1 2 7 
 − 
4 −1 1 −1 1 4

   
(0 + 3) −(0 + 1) (3 − 4) 3 −1 −1
   
= −(0 − 3) (0 − 1) −(6 − 7) = 3 −1
   
1
   
(−7 − 4) −(−2 − 1) (8 − 7) −11 3 1

132
Now the adjoint matrix is the transpose of cofactor matrix, i.e.
 
3 3 −11
 
adjA = C T = −1 −1
 
3 
 
−1 1 1

Now


2 1 1

det A = 1 −1

1


1 0 0

C2 − 3C1

Expanding along C1


1 1
= 1 ×
−1

1

= −1 − 1 = −2.

Now to find inverse,


   
3 3 −11 −3 −3 11
1 1   1 
A−1 = (adjA) = −1 −1 3 =  1 −3 ,
   
1
det A −2   2 
−1 1 1 1 −1 −1

hence we still obtain the same result.

Tutorials 5.4.6. By using the methods below, find the inverse of matrices
below if they exist:

(i) Gauss-Jordan Elimination

(ii) Adjoint Method


 
−3 6
(a) A =  
4 5
 
−1 3
(b) B =  
3 −2

133
 
3 4 −1
 
(c) C = 1 0 3 
 
 
2 5 −4
 
−1 3 −4
 
(d) D =  2 4 1 
 
 
−4 2 −9

5.5 System of Linear equations: Matrix method


System of linear equation plays a vital role in the subject of linear algebra.
Many problems in linear algebra reduces to finding the solution of a system
of linear equations. The system of linear equations involve constants (scalars)
which may come from any field. In this chapter all our constants (scalars) will
come from the field of real numbers.

Definition 5.5.1. A linear equation over a field of real numbers with unknowns
x1 , x2 , . . . , xn is an equation that can be put in the standard form

a1 x1 + a2 x2 + . . . + an xn = b

where a1 , a2 , . . . an , and b are constants (scalars) which comes from the field of
real numbers (R) and x1 , x2 , . . . , xn are variables. The constant ak is called the
coefficient of xk , and b is called the constant term of the equation.
A system of linear equations (or linear system) is a finite collection of
linear equations in same variables. For instance, a linear system of m equations
in n variables x1 , x2 , . . . , xn can be written as:

a11 x1 + a12 x2 + · · · + a1n x = b1

a21 x1 + a22 x2 + · · · + a2n x = b2


(5.1)
.. ..
. .

am1 x1 + am2 x2 + · · · + amn xn = bm

134
Definition 5.5.2. A system of equations is consistent if it has at least one
solution and it is inconsistent if it has no solution.

Theorem 5.5.3. Given a system of linear equations in several unknowns, then


the system of linear equations will have one of the following:

(a) A unique (only one) solution (consistent)

(b) Infinitely many solutions

(c) No solution

In previous year, you were introduced to elimination method (simultaneous


equation) as a way of solving system of linear equations. We will now use the
matrix method to solve the system of linear equations.

5.5.1 Matrix Method

Gauss-Jordan Elimination Method

Example 5.5.4. Solve for x, y and z using the Gauss-Jordan elimination method
of the following system of linear equations.

2x + 7y + z = 4

x + 4y − z = 9

x + 3y = 3

   
2 7 1 4
   
A = 1 −1 ; B = 9
   
4
   
1 3 0 3
 

2 7 1 4
 
(A|B) = 1 4 −1 9
 
 

1 3 0 3

135
First interchange row 1 and 2.

 
−1 9 R2 ↔ R1

1 4
 
∼ 2
 
7 1 4
 

1 3 0 3
 
4 −1 9

1
 
∼ 0 −1 3 −14 R2 − 2R1
 
 
−1 1 −6 R3 − R1

0
 
4 −1 9

1
 
∼ 0 1 −3 14  −R2
 
 
−1 1 −6

0
 
0 11 −47 R1 − 4R2

1
 
∼ 0 1 −3 14 
 
 
0 −2 8

0 R3 + R2
 
0 11 −47

1
 
∼ 0 1 −3 14 
 
 
0 1 −4 − 21 R3

0
 
0 0 −3 R1 − 11R3

1
 
∼ 0
 
1 0 2  R2 + 3R3
 
0 1 −4

0

∴ the system has unique solution with solutions x = −3, y = 2 and z = −4.

Example 5.5.5. Solve for x, y and z using the Gauss-Jordan elimination method
of the following system of linear equations.

2x − 3y + z = 5

−x + 2y − z = 0

−x + 3y − 2z = 4

136
 
−3

2 1 5
 
(A|B) = −1 −1 0
 
2
 
−1 −2 4

3

First interchange row 1 and 2.

 
−1 2 −1 0 R1 ↔ R2

 
∼  2 −3 1 5
 
 
−1 3 −2 4

 
1 −2 1 0 −R1

 
∼  2 −3 1 5
 
 
−1 3 −2 4

 
1 −2 1 0

 
∼ 0 1 −1 5 R2 − 2R1
 
 
0 1 −1 4 R3 − R1

 
1 0 −1 10 R1 + 2R2

 
∼ 0 1 −1 5  R3 − R1
 
 
0 0 0 −1

From row 3, we see that we have equation 0x+0y +0z = −1 which is impossible;
hence the system will have a no solution.

Example 5.5.6. Solve for x, y and z using the Gauss-Jordan elimination method
of the following system of linear equations.

x − 3y + 2z = −1

2x − 5y − z = 2

2x − 7y + 9z = −6

137
   
1 −3 2 −1
   
A = 2 −5 −1 ; B =  2 
   
   
2 −7 9 −6

 
−3 2 −1

1
 
(A|B) = 2 −5 −1 2 
 
 
−7 9 −6

2
 
4 −1 9

1
 
∼ 0 −1 3 −14 R2 − 2R1
 
 
−1 1 −6 R3 − 2R1

0
 
−3 2 −1

1
 
∼ 0 1 −5 4  −R2
 
 
−1 5 −4

0
 
0 −13 11 R1 + 3R2

1
 
∼ 0 1 −5 4 
 
 

0 0 0 0 R3 + R1

∴ the system has infinitely many solutions.


If we look in rows one and two, we see that coefficients of x and y are 1, we
say x and y are leading variables. The remaining variable z is called free
vaiable. We then solve the equation in terms of the free variable z.
From row 1, we get x = 13z + 11 and from row 2, we get y = 5z + 4.
From these we can see that we can treat z as a parameter, hence we let z = t,
and our solution will be in parametric equation as:
x = 13t + 11, y = 5t + 4 and z = t.

138
Inverse of Matrix Method

Matrix inverses can be used to solve certain systems of linear equations. The
system of linear equations can be written as a single matrix equation

AX = B.

where A and B are known matrices and X is to be determined. If A is invertible,


we multiply each side of the equation on the left by A−1 to get
A−1 AX = A−1 B, then In X = A−1 B, hence X = A−1 B. This will yield the
unique solution

Theorem 5.5.7. Suppose a system of n equations in n variables is written in


matrix form as AX = B. If the n × n coefficient matrix A is invertible, the
system has the unique solution X = A−1 B.

We will verify this method again using the previous examples.

Example 5.5.8.

2x + 7y + z = 4

x + 4y − z = 9

x + 3y = 3
    
2 7 1 x 4
    
We can express as AX = B as follows 1 −1 y  = 9 .
    
4
    
1 3 0 z 3
We have already seen that
 
−3 −3 11
 2 2 2 
A−1 =  1 1 −3 
.

 2 2 2 
1 −1 −1
2 2 2

139
Using inverse of matrix method to solve X = A−1 B, i.e.
    
−3 −3 11
x 2 2 2 4
    
   1
y  =  2 1 −3  
2 2  9
 
  
1 −1 −1
z 2 2 2 3
 
( −3 )(4) + ( −3
)(9) + ( 11
)(3)
 2 2 2 
=  ( 12 )(4) + ( 12 ) + ( −3
 
2 )(3) 
 
( 12 )(4) + ( −1
2 )(9) + ( −1
2 )(3)
 
27 33
−6 − 2 + 2
 
=  2 + 92 − 29 
 
 
2 − 92 − 23
 
−3
 
=  2 .
 
 
−4

Hence x = −3, y = 2 and z = −4.

Cramer’s Rule

This method uses determinant to solve the system of linear equations.


Consider the system of linear equation AX = B with unique solution. Then

• det A exist, say | A |= D.

• Let Dx =| Ax | where Ax is a matrix obtained by replacing the column of


coefficients of x by B.
Similarly Dy =| Ay | where Ay is a matrix obtained by replacing the
column of coefficients of y by B; Dy =| Ay | in similar fashion, etc.

• Then we can obtain unique solution

Dx Dy Dz
x= ; y= ; z= ; etc. where D 6= 0.
D D D

140
Example 5.5.9.

2x + 7y + z = 4

x + 4y − z = 9

x + 3y = 3
    
2 7 1 x 4
    
−1 y  = 9 .
    
1 4
    
1 3 0 z 3
Then


2 7 1

D =| A | = 1 −1

4


1 3 0


2 7 1

= 3

11 0


1 3 0


3 11
= 1 × = 3 × 3 − 11 × 1 = 9 − 11 = −2.

1 3



4 7 1

Dx =| Ax | = 9 4 −1



3 3 0


4 7 1

= 13 11 0



3 3 0


13 11
=1× = 13 × 3 − 11 × 3 = 39 − 33 = 6.

3 3

141


2 4 1

Dy =| Ay | = 1 −1

9


1 3 0


2 4 1

= 3

13 0


1 3 0


3 13
= 1 × = 3 × 3 − 13 × 1 = 9 − 13 = −4.

1 3



2 7 4

Dz =| Az | = 1

4 9


1 3 3

−2

0 1

= 1

4 9

−1 −6

0

−2

1
= −1 × = −[1 × (−6) − (−1) × (−2)] = −(−6 − 2) = 8.
−1 −6

So

Dx 6
x= = = −3,
D −2
Dy −4
y= = = 2,
D −2
Dz 8
z= = = −4.
D −2

5.6 3 × 3 systems with parameters


Consider the system of linear equations:

142
Example 5.6.1.

x + 3y + (2 − 3k)z = 3

2x + 7y + (4 − 7k)z = 7

2x + ky + (k + 2)z = 2k − 1

Determine the values of k ∈ R such that

(a) The system has no solution

(b) The system has infinitely many solutions

(c) The system has unique (only one) solution (consistent).

Solution:

We perform Gauss-Jordan elimination of the augmented matrix to obtain:


 
1 3 2 − 3k 3

 
2 7 4 − 7k 7 
 
 
2 k k + 2 2k − 1

 
2 − 3k 3

1 3
 
∼ 0 −k 1  R2 − 2R1
 
1
 
0 k − 6 7k − 2 2k − 7 R3 − 2R1

 

1 0 2 0
  R1 − 3R2
∼ 0 −k
 
1 1 
 
k2 + k − 2 k − 1

0 0
R3 − (k − 6)R2
 

1 0 2 0
 
∼ 0 1 −k
 
1 
 
(k + 2)(k − 1) k − 1

0 0

Then

(a) The system has no solution if


(k + 2)(k − 1) = 0 and k − 1 6= 0, so k = −2 and k 6= 1, since for k = −2,

143
we obtain  

1 0 2 0
 
 
0 1 2 1 
 
0 −3

0 0
which is clearly impossible.

(b) The system has infinitely many solutions if


(k + 2)(k − 1) = 0 and k − 1 = 0, thus k = 1.

(c) The system has unique solution if


(k + 2)(k − 1) 6= 0 thus k 6= −2 and k 6= 1.

Tutorials 5.6.2. 1. Use Gauss-Jordan Elimination to find values of x, y


and z where possible and describe whether each type of solution is either
unique, no solution or infinitely many solutions.

(a)

x + 3y + 2z = 4

−2x − 7y + z = 9

3x + 10y + 9z = 19

(b)

x + 2y + z = 2

2x + 3y + 3z = 3

−3x − 4y − 5z = −5

(c)

x + 2y + 3z = 3

2x + 3y + 8z = 4

5x + 8y + 19z = 11

(d) From (a), (b) and (c) above, where you found a unique solution, use
the inverse of matrix method to confirm the solution.

144
2. In each of the following solve for x, y and z using

(i) Inverse of matrix method,

(ii) Cramer’s Rule

(a)

x + 2y + 3z = 3

2x + 3y + 8z = 4

5x + 8y + 19z = 11

(b)

x + y + 2z = 8

−x − 2y + 3z = 1

3x − 7y + 4z = 10

(c)

2x + 5y − z = −4

x + 2y + z = 3

3x − 2y − z = 5

3. For which value(s) of k will the following system of linear equations have

(i) No solution

(ii) Infinitely many solutions

(iii) Unique solution

(a)

x + 2y + z = 3

ky + 5z = 10

2x + 7y + kz = 4

145
(b)

y + 2kz = 0

x + 2y + 6z = 2

kx + 2z = 1

4. Find values of k and a such that the following system has

(i) No solution

(ii) Infinitely many solutions

(iii) Unique solution

x + 2y + 2z = 1

x + ky + 3z = 3

x + 11y + kz = a

5.6.1 Elementary Matrices and Test for Invertibility

Here we are going to look at a very important theorem which determines whether
a matrix is invertible or not.

Definition 5.6.3. An elementary matrix is an n × n matrix which can be


obtained from the identity matrix In by performing on In a single elementary
row transformation.

Theorem 5.6.4. If R is the reduced row echelon form of an matrix A, then


either R has a row of zeros or R is the identity matrix In .

Theorem 5.6.5. If A is an matrix, then the following are equivalent:

(a) A is invertible.

(b) AX = 0 has only the trivial solution.

(c) The reduced row echelon form of A is In .

146
(d) A is expressible as a product of elementary matrices.

(e) AX = B is consistent for every matrix B.

(f ) AX = B has exactly one solution for every matrix B.

Theorem 5.6.6. Let E be an elementary matrix.

(a) If E results from multiplying a row of In by a nonzero number k, then


det(E) = k.

(b) If E results from interchanging two rows of In , then det(E) = −1.

(c) If E results from adding a multiple of one row of to In another, then


det(E) = 1.

Remarks 5.6.7. If B is an n×n matrix and E1 , E2 , . . . , Er are n×n elementary


matrices, then

det(E1 E2 . . . Er B) = det(E1 ) det(E2 ) . . . det(Er ) det(B).

Theorem 5.6.8. A square matrix A is invertible if and only if det(A) 6= 0.

Proof. (⇒:) Assume A is invertible, i.e. A−1 exists. Then AA−1 = In and

1 = det(In ) = det(AA−1 ) = det(A) det(A−1 ).

From this it is clear that det(A) 6= 0.


(⇐:) Conversely, assume det(A) 6= 0. Let R be the reduced row echelon form
of A. Then there exists elementary matrices E1 , E2 , . . . , Er such that

R = E1 E2 . . . Er A

and from Remark 5.6.7,

det(R) = det(E1 ) det(E2 ) . . . det(Er ) det(A). (5.2)

Now by Theorem (5.6.6) the determinant of an elementary matrix is non-zero,


and since det(A) 6= 0, it then follows that det(R) 6= 0, which imply that R
cannot have a row of zeros. Then by Theorem (5.6.4), R = In and hence A is
invertible by Theorem (5.6.5). 

147
Tutorials 5.6.9. 1. Which of the following matrices are elementary matri-
ces?
 
1 0 −8
 
(a)
 
0 1 0 
 
0 0 1
 
1 0 0
 √ 
(b)  0 − 7 0
 
 
−3 0 1
 
1 0 0 0
 
 
0 0 1 0
(c) 



0 1 0 0
 
0 0 0 1
 
1 0
(d)  
0 − 23
 
1 1 0
 
(e)
 
0 1 0
 
0 0 0

2. From question 5. above, show which operations where used to obtain


those matrices which are elementary matrices.

“If others would think as hard as I did, then they would get similar results.”
Isaac Newton

148

Das könnte Ihnen auch gefallen